You are on page 1of 55

Reviewer

RESPIRATORY SYSTEM C. Chest pain


D. Bulging fontanel
1. Betty is a 9-year-old girl diagnosed with
cystic fibrosis. Which of the following must 7. Baby Melody is a neonate who has a very
Nurse Archie keep in mind when developing a low-birth-weight. Nurse Josie carefully
care plan for the child? monitors inspiratory pressure and oxygen (O2)
concentration to prevent which of the
A. Pulmonary secretions are abnormally thick. following?
B. Elevated levels of potassium are found in the
sweat. A. Meconium aspiration syndrome
C. CF is an autosomal dominant hereditary B. Bronchopulmonary dysplasia (BPD)
disorder. C. Respiratory syncytial virus (RSV)
D. Obstruction of the endocrine glands occurs. D. Respiratory distress syndrome (RDS)

2. Veronica’s parents were told that their 8. Archie who weighs 44 lb has been given an
daughter needs ribavirin (Virazole). This drug is order for amoxicillin 500 mg b.i.d. The drug text
used to treat which of the following? notes that the daily dose of amoxicillin is 50
mg/kg/day in two divided doses. What dose in
A. Cystic fibrosis milligrams is safest for this child?
B. Otitis media
C. Respiratory syncytial virus (RSV) A. 1000 mg
D. Bronchitis B. 750 mg
C. 500 mg
3. Beta-adrenergic agonists such
D. 250 mg
as albuterol are given to Reggie, a child
with asthma. Such drugs are administered 9. The Andrews family has been taking good
primarily to do which of the following? care of their youngest, Archie, who was
diagnosed with asthma. Which of the following
A. Dilate the bronchioles statements indicate a need for further home
B. Reduce secondary infections care teaching?
C. Decrease postnasal drip
D. Reduce airway inflammation A. “He should increase his fluid intake regularly
to thin secretions.”
4. Alice is rushed to the emergency B. “We’ll make sure that he avoids exercise to
department during an acute, severe prolonged prevent attacks.”
asthma attack and is unresponsive to usual C. “He is to use his bronchodilator inhaler
treatment. The condition is referred to as before the steroid inhaler.”
which of the following? D. “We need to identify what things trigger his
A. Status asthmaticus attacks.”
B. Reactive airway disease 10. Which of the following instructions should
C. Intrinsic asthma Nurse Cheryl include in her teaching plan for
D. Extrinsic asthma the parents of Reggie with otitis media?
5. Which of the following infants is least A. Placing the child in the supine position to
probable to develop sudden infant death bottle-feed
syndrome (SIDS)? B. Giving prescribed amoxicillin (Amoxil) on an
A. Baby Angela who was premature empty stomach
B. A sibling of Baby Angie who died of SIDS C. Cleaning the inside of the ear canals with
C. Baby Gabriel with prenatal drug exposure cotton swabs
D. Baby Gabby who sleeps on his back D. Avoiding contact with people who have
upper respiratory tract infections
6. Fred is a 12-year-old boy diagnosed with
pneumococcal pneumonia. Which of the 11. Immunization of children
following would Nurse Nica expect to assess? with Haemophilus influenzae type B (Hib)
vaccine decreases the incidence of which of
A. Mild cough the following conditions?
B. Slight fever
Reviewer

A. Bronchiolitis disorder concerning the exocrine, not endocrine


B. Laryngotracheobronchitis (LTB) glands.
C. Epiglottitis
D. Pneumonia D: The thick mucus blocks the exocrine glands.

12. Which of the following respiratory 2. Answer: C. Respiratory syncytial virus (RSV)
conditions is always considered a medical C: Ribavirin is an antiviral medication used for
emergency? treating RSV infection and for children with RSV
A. Asthma who are compromised (such as children with
B. Cystic fibrosis (CF) bronchopulmonary dysplasia or heart disease).
C. Epiglottiditis A,B,D: The drug is not used to treat bronchiolitis,
D. Laryngotracheobronchitis (LTB) otitis media, or CF.
13. When assessing a child’s cultural 3. Answer: A. Dilate the bronchioles
background, the nurse in charge should keep in
mind that: A: Beta-adrenergic agonists, such as albuterol,
are highly effective bronchodilators and are
A. Heritage dictates a group’s shared values used to dilate the narrow airways associated
B. Physical characteristics mark the child as part with asthma.
of a particular culture
C. Cultural background usually has little bearing B: Antibiotics are used to prevent secondary
on a family’s health practices infection.
D. Behavioral patterns are passed from one
C: Decongestants may be given to decrease
generation to the next
postnasal drip.
14. Which of the following is the best method
D: Corticosteroids may be used for their anti-
for performing a physical examination on a
inflammatory effect.
toddler
4. Answer: A. Status asthmaticus
A. From head to toe
B. Distally to proximally A: Status asthmaticus is an acute, prolonged,
C. From abdomen to toes, the to head severe asthma attack that is unresponsive to
D. From least to most intrusive usual treatment. Typically, the child requires
hospitalization.
15. Nurse Veronica is teaching a group of
parents about otitis media. When discussing B: Reactive airway disease is another general
why children are predisposed to this disorder, term for asthma.
the nurse should mention the significance of
which anatomical feature? C: Intrinsic is a term used to denote internal
precipitating factors, such as viruses.
A. Nasopharynx
B. Eustachian tubes D: Extrinsic is a term used to denote external
C. External ear canal precipitating factors, such as allergens.
D. Tympanic membrane
5. Answer: D. Baby Gabby who sleeps on his
Answers and Rationale back

1. Answer: A. Pulmonary secretions are D: Infants who sleep on their back are least
abnormally thick. likely to develop SIDS. However, SIDS has been
associated with infants who sleep on their
A: CF is identified by abnormally thick abdomens.
pulmonary secretions.
A,B,C: Being premature, having a sibling who
B: Diagnosis of CF is based on elevated chloride died of SIDS, and being prenatally exposed to
levels detected in sweat. drugs all place the infant at high risk for
developing SIDS.
C: It is a chronic, inherited disorder, particularly
an autosomal recessive hereditary 6. Answer: C. Chest pain
Reviewer

C: Older children with pneumococcal children prone to otitis should avoid people
pneumonia may complain of chest pain. known to have an upper respiratory tract
infection.
A,B: A mild cough and slight fever are
commonly assessed with viral pneumonia. A: A bottle-fed child should be fed in an upright
position because feeding the child in the supine
D: A bulging fontanel may be seen in infants position may actually precipitate otitis by
with meningitis or increased intracranial allowing the formula to pool in the pharyngeal
pressure. cavity.
7. Answer: B. Bronchopulmonary dysplasia B: Amoxicillin, when prescribed, should be given
(BPD) with food to prevent stomach upset.
B: Close monitoring of inspiratory pressure and C: Cotton swabs can cause injuries such as
O2 concentration is necessary to prevent BPD, tympanic perforation. They may be used to
which is related to the use of high inspiratory clean the outer ear, but they should never be
pressures and O2concentrations especially in inserted into the ear canal.
very low-birth-weight and extremely low-birth-
weight neonates with lung disorders. 11. Answer: C. Epiglottitis

A: Meconium aspiration syndrome is a respiratory C: Epiglottitis is a bacterial infection of


disorder created by the aspiration of meconium the epiglottis primarily caused by Hib.
in perinatal period. Administration of the vaccine has decreased the
incidence of epiglottitis.
C: RSV is a group of viruses that cause
respiratory tract infections, such as bronchiolitis A: Bronchiolitis is usually caused by Respiratory
and pneumonia. Syncytial Virus (RSV).

D: RDS, a disorder caused by lack of surfactant, B: Acute LTB is of viral origin.


usually is found in premature neonates.
D: The most common bacterial organisms
8. Answer: C. 500 mg causing pneumonia in children are
pneumococci, streptococci, and staphylococci.
C: First, calculate the child’s weight in kg: 44/2.2
= 20 kg. Then calculate the appropriate daily 12. Answer: C. Epiglottiditis
dose according to the drug text: 50 mg/kg/day =
50 mg x 20 kg = 1,000 mg/day. the abbreviation C: Epiglottiditis, acute and severe inflammation
b.i.d. means twice daily; therefore 1,000 divided of the epiglottis, is always considered an acute
medical emergency because it can lead to
by 2 equals 500 mg.
acute, life-threatening airway obstruction.
9. Answer: B. “We’ll make sure that he avoids
exercise to prevent attacks.” A: Asthma is a chronic disease; however, status
asthmaticus and acute attacks require prompt
B: Additional teaching is needed if the family treatment.
states that the child with asthma should avoid
exercise to prevent attacks. Children with B: CF is a chronic disease and is not considered
asthma should be encouraged to exercise as an emergency.
tolerated. D: Acute LTB requires close observation for
A,C,D: Identifying triggers, using a airway obstruction, but this condition is not
bronchodilator inhaler before a steroid inhaler, always an emergency.
and increasing fluid intake are appropriate 13. Answer: D. Behavioral patterns are passed
measures to be included in a home care from one generation to the next
teaching program for the child with asthma and
his family. D: A family’s behavioral patterns and values are
passed from one generation to the next.
10. Answer: D. Avoiding contact with people
who have upper respiratory tract infections A: Although heritage plays a role in culture, it
does not dictate a group’s shared values and its
D: Otitis media is commonly precipitated by an
upper respiratory tract infection. Therefore,
Reviewer

effect on culture is weaker than that of tracheostomy tube, the nurse initially should
behavioral patterns. plug the opening in the tube for:

B: Physical characteristics do not indicate a A. 15 to 60 seconds.


child’s culture. B. 5 to 20 minutes.
C. 30 to 40 minutes.
C: Cultural background commonly plays a major
D. 45 to 60 minutes.
role in determining a family’s health practices.
4. Gina, a home health nurse is visiting a home
14. Answer: Answer: D. From least to most care client with advanced lung cancer. Upon
intrusive assessing the client, the nurse discovers
D: When examining a toddler or any small child, wheezing, bradycardia, and a respiratory rate
the best way to perform the exam is from least of 10 breaths/minute. These signs are
to most intrusive. associated with which condition?

A: Starting at the head or abdomen is intrusive A. Hypoxia


and should be avoided. B. Delirium
C. Hyperventilation
B: Proceeding from distal to proximal is D. Semiconsciousness
inappropriate at any age.
5. A male client with Guillain-Barré syndrome
15. Answer: B. Eustachian tubes develops respiratory acidosis as a result of
reduced alveolar ventilation. Which
B: In a child, Eustachian tubes are short and lie
combination of arterial blood gas (ABG) values
in a horizontal plane, promoting entry of
confirms respiratory acidosis?
nasopharyngeal secretions into the tubes and
thus setting the stage for otitis media. A. pH, 5.0; PaCO2 30 mm Hg
B. pH, 7.40; PaCO2 35 mm Hg
A,C,D: The nasopharynx, tympanic membrane,
C. pH, 7.35; PaCO2 40 mm Hg
external ear canal have no unusual features that
D. pH, 7.25; PaCO2 50 mm Hg
would predispose a child to otitis media.
6. A female client with interstitial lung disease
is prescribed prednisone (Deltasone) to control
1. The nurse is caring for a male client with a inflammation. During client teaching, the nurse
chest tube. If the chest drainage system is stresses the importance of taking prednisone
accidentally disconnected, what should the exactly as prescribed and cautions against
nurse plan to do? discontinuing the drug abruptly. A client who
discontinues prednisone abruptly may
A. Place the end of the chest tube in a experience:
container of sterile saline.
B. Apply an occlusive dressing and notify A. hyperglycemia and glycosuria.
the physician. B. acute adrenocortical insufficiency.
C. Clamp the chest tube immediately. C. GI bleeding.
D. Secure the chest tube with tape. D. restlessness and seizures.

2. A male elderly client is admitted to an acute


care facility with influenza. The nurse monitors
7. A male client is admitted to the health care
the client closely for complications. What is the
facility for treatment of chronic obstructive
most common complication of influenza?
pulmonary disease. Which nursing diagnosis is
A. Septicemia most important for this client?
B. Pneumonia
A. Activity intolerance related to fatigue
C. Meningitis
B. Anxiety related to actual threat to
D. Pulmonary edema
health status
3. A female client has a tracheostomy but C. Risk for infection related to retained
doesn’t require continuous mechanical secretions
ventilation. When weaning the client from the D. Impaired gas exchange related to
airflow obstruction
Reviewer

8. A male client abruptly sits up in bed, reports 13. The amount of air inspired and expired
having difficulty breathing and has an arterial with each breath is called:
oxygen saturation of 88%. Which mode of
oxygen delivery would most likely reverse the A. tidal volume.
B. residual volume.
manifestations?
C. vital capacity.
A. Simple mask D. dead-space volume.
B. Non-rebreather mask
C. Face tent 14. A male client with pneumonia develops
respiratory failure and has a partial pressure of
D. Nasal cannula
arterial oxygen of 55 mm Hg. He’s placed on
9. A male adult client with cystic fibrosis is mechanical ventilation with a fraction of
admitted to an acute care facility with an acute inspired oxygen (FIO2) of 0.9. The nursing goal
respiratory infection. Prescribed respiratory should be to reduce the FIO2 to no greater
treatment includes chest physiotherapy. When than:
should the nurse perform this procedure?
A. 0.21
A. Immediately before a meal B. 0.35
B. At least 2 hours after a meal C. 0.5
C. When bronchospasms occur D. 0.7
D. When secretions have mobilized
15. Nurse Mickey is administering a purified
10. On arrival at the intensive care unit, a protein derivative (PPD) test to a homeless
critically ill female client suffers respiratory client. Which of the following statements
arrest and is placed on mechanical ventilation. concerning PPD testing is true?
The physician orders pulse oximetry to
monitor the client’s arterial oxygen saturation A. A positive reaction indicates that the
(SaO2) noninvasively. Which vital sign client has active tuberculosis (TB).
B. A positive reaction indicates that the
abnormality may alter pulse oximetry values?
client has been exposed to the disease.
A. Fever C. A negative reaction always excludes the
B. Tachypnea diagnosis of TB.
C. Tachycardia D. The PPD can be read within 12 hours
D. Hypotension after the injection.

11. The nurse is caring for a male client who 16. Nurse Murphy administers albuterol
recently underwent a tracheostomy. The first (Proventil), as prescribed, to a client with
priority when caring for a client with a emphysema. Which finding indicates that the
tracheostomy is: drug is producing a therapeutic effect?

A. helping him communicate. A. Respiratory rate of 22 breaths/minute


B. keeping his airway patent. B. Dilated and reactive pupils
C. encouraging him to perform activities of C. Urine output of 40 ml/hour
daily living. D. Heart rate of 100 beats/minute
D. preventing him from developing an
infection. 17. What is the normal pH range for arterial
blood?
12. For a male client with chronic obstructive
pulmonary disease, which nursing intervention A. 7 to 7.49
B. 7.35 to 7.45
would help maintain a patent airway?
C. 7.50 to 7.60
A. Restricting fluid intake to 1,000 ml/day D. 7.55 to 7.65
B. Enforcing absolute bed rest
C. Teaching the client how to perform 18. Before weaning a male client from a
ventilator, which assessment parameter is
controlled coughing
D. Administering prescribed sedatives most important for the nurse to review?
regularly and in large amounts
Reviewer

A. Fluid intake for the last 24 hours 23. Nurse Eve formulates a nursing diagnosis of
B. Baseline arterial blood gas (ABG) levels Activity intolerance related to inadequate
C. Prior outcomes of weaning oxygenation and dyspnea for a client with
D. Electrocardiogram (ECG) results chronic bronchitis. To minimize this problem,
the nurse instructs the client to avoid
19. Which of the following would be most conditions that increase oxygen demands.
appropriate for a male client with an arterial Such conditions include:
blood gas (ABG) of pH 7.5, PaCO2 26 mm Hg,
O2 saturation 96%, HCO3 24 mEq/L, and PaO2 A. drinking more than 1,500 ml of fluid
94 mm Hg? daily.
B. being overweight.
A. Administer a prescribed decongestant. C. eating a high-protein snack at bedtime.
B. Instruct the client to breathe into a D. eating more than three large meals a
paper bag.
day.
C. Offer the client fluids frequently.
D. Administer prescribed supplemental 24. A black male client with asthma seeks
oxygen. emergency care for acute respiratory distress.
Because of this client’s dark skin, the nurse
20. A female client is receiving supplemental
should assess for cyanosis by inspecting the:
oxygen. When determining the effectiveness
of oxygen therapy, which arterial blood gas A. lips.
value is most important? B. mucous membranes.
C. nail beds.
A. pH
D. earlobes.
B. Bicarbonate (HCO3–)
C. Partial pressure of arterial oxygen 25. A female client with asthma is receiving a
(PaO2) theophylline preparation to promote
D. Partial pressure of arterial carbon bronchodilation. Because of the risk of drug
dioxide (PaCO2) toxicity, the nurse must monitor the client’s
serum theophylline level closely. The nurse
21. Nurse Julia is caring for a client who has a knows that the therapeutic theophylline
tracheostomy and temperature of 103° F (39.4°
concentration falls within which range?
C). Which of the following interventions will
most likely lower the client’s arterial blood A. 1 to 2 mcg/ml
oxygen saturation? B. 2 to 5 mcg/ml
C. 5 to 10 mcg/ml
A. Endotracheal suctioning
D. 10 to 20 mcg/ml
B. Encouragement of coughing
C. Use of cooling blanket 26. A male client is to receive I.V. vancomycin
D. Incentive spirometry (Vancocin). When preparing to administer this
drug, the nurse should keep in mind that:
22. For a male client who has a chest tube
connected to a closed water-seal drainage A. vancomycin should be infused over 60
system, the nurse should include which action to 90 minutes in a large volume of fluid.
in the plan of care? B. vancomycin may cause irreversible
neutropenia.
A. Measuring and documenting the C. vancomycin should be administered
drainage in the collection chamber rapidly in a large volume of fluid.
B. Maintaining continuous bubbling in the D. vancomycin should be administered
water-seal chamber
over 1 to 2 minutes as an I.V. bolus.
C. Keeping the collection chamber at chest
level 27. Before seeing a newly assigned female
D. Stripping the chest tube every hour client with respiratory alkalosis, the nurse
quickly reviews the client’s medical history.
Which condition is a predisposing factor for
respiratory alkalosis?
Reviewer

A. Myasthenia gravis prevent it from being disconnected, rather than


B. Type 1 diabetes mellitus taping it after it has been disconnected.
C. Extreme anxiety
D. Narcotic overdose Answer B. Pneumonia is the most common
complication of influenza. It may be either
28. At 11 p.m., a male client is admitted to the primary influenza viral pneumonia or
emergency department. He has a respiratory pneumonia secondary to a bacterial infection.
rate of 44 breaths/minute. He’s anxious, and Other complications of influenza include
wheezes are audible. The client is immediately myositis, exacerbation of chronic obstructive
given oxygen by face mask and pulmonary disease, and Reye’s syndrome.
methylprednisolone (Depo-medrol) I.V. At Myocarditis, pericarditis, transverse myelitis,
11:30 p.m., the client’s arterial blood oxygen and encephalitis are rare complications of
saturation is 86% and he’s still wheezing. The influenza. Although septicemia may arise when
nurse should plan to administer: any infection becomes overwhelming, it rarely
results from influenza. Meningitis and
A. alprazolam (Xanax). pulmonary edema aren’t associated with
B. propranolol (Inderal)
influenza.
C. morphine.
D. albuterol (Proventil)
Answer B. Initially, the nurse should plug the
29. Pulmonary disease (COPD), which nursing opening in the tracheostomy tube for 5 to 20
action best promotes adequate gas exchange? minutes, and then gradually lengthen this
A. Encouraging the client to drink three interval according to the client’s respiratory
glasses of fluid daily status. A client who doesn’t require continuous
B. Keeping the client in semi-Fowler’s mechanical ventilation already is breathing
position without assistance, at least for short periods;
C. Using a high-flow Venturi mask to therefore, plugging the opening of the tube for
deliver oxygen as prescribed only 15 to 60 seconds wouldn’t be long enough
D. Administering a sedative as prescribed to reveal the client’s true tolerance to the
procedure. Plugging the opening for more than
30. Nurse Joana is teaching a client with 20 minutes would increase the risk of acute
emphysema how to perform pursed-lip respiratory distress because the client requires
breathing. The client asks the nurse to explain an adjustment period to start breathing
the purpose of this breathing technique. Which normally.
explanation should the nurse provide?

A. It helps prevent early airway collapse. Answer A. As the respiratory center in the brain
B. It increases inspiratory muscle strength becomes depressed, hypoxia occurs, producing
C. It decreases use of accessory breathing wheezing, bradycardia, and a decreased
muscles. respiratory rate. Delirium is a state of mental
D. It prolongs the inspiratory phase of confusion characterized by disorientation to
respiration. time and place. Hyperventilation (respiratory
rate greater than that metabolically necessary
Answers and Rationales
for gas exchange) is marked by an increased
Answer A. If a chest drainage system is respiratory rate or tidal volume, or both.
disconnected, the nurse may place the end of Semiconsciousness is a state of impaired
the chest tube in a container of sterile saline or consciousness characterized by limited motor
water to prevent air from entering the chest and verbal responses and decreased
tube, thereby preventing negative respiratory orientation.
pressure. The nurse should apply an occlusive
dressing if the chest tube is pulled out — not if
Answer D. In respiratory acidosis, ABG analysis
the system is disconnected. The nurse shouldn’t
reveals an arterial pH below 7.35 and partial
clamp the chest tube because clamping
pressure of arterial carbon dioxide (PaCO2)
increases the risk of tension pneumothorax. The
above 45 mm Hg. Therefore, the combination of
nurse should tape the chest tube securely to
a pH value of 7.25 and a PaCO2 value of 50 mm
Hg confirms respiratory acidosis. A pH value of
Reviewer

5.0 with a PaCO2 value of 30 mm Hg indicates values directly.


respiratory alkalosis. Options B and C represent
normal ABG values, reflecting normal gas
exchange in the lungs. Answer B. Maintaining a patent airway is the
most basic and critical human need. All other
interventions are important to the client’s well-
Answer B. Administration of a corticosteroid being but not as important as having sufficient
such as prednisone suppresses the body’s oxygen to breathe.
natural cortisol secretion, which may take
weeks or months to normalize after drug
discontinuation. Abruptly discontinuing such Answer C. Controlled coughing helps maintain a
therapy may cause the serum cortisol level to patent airway by helping to mobilize and
drop low enough to trigger acute adrenocortical remove secretions. A moderate fluid intake
insufficiency. Hyperglycemia, glycosuria, GI (usually 2 L or more daily) and moderate activity
bleeding, restlessness, and seizures are help liquefy and mobilize secretions. Bed rest
common adverse effects of corticosteroid and sedatives may limit the client’s ability to
maintain a patent airway, causing a high risk of
therapy, not its sudden cessation.
infection from pooled secretions.

Answer D. A patent airway and an adequate


breathing pattern are the top priority for any Answer A. Tidal volume is the amount of air
client, making impaired gas exchange related to inspired and expired with each breath. Residual
airflow obstruction the most important nursing volume is the amount of air remaining in the
diagnosis. The other options also may apply to lungs after forcibly exhaling. Vital capacity is the
this client but are less important. maximum amount of air that can be moved out
of the lungs after maximal inspiration and
expiration. Dead-space volume is the amount of
Answer B. A non-rebreather mask can deliver air remaining in the upper airways that never
levels of the fraction of inspired oxygen (FIO2) reaches the alveoli. In pathologic conditions,
as high as 100%. Other modes — simple mask, dead space may also exist in the lower airways.
face tent and nasal cannula — deliver lower
levels of FIO2.
Answer C. An FO2 greater than 0.5 for as little
as 16 to 24 hours can be toxic and can lead to
Answer B. The nurse should perform chest decreased gas diffusion and surfactant activity.
physiotherapy at least 2 hours after a meal to The ideal oxygen source is room air F IO 2 0.18
reduce the risk of vomiting and aspiration. to 0.21.
Performing it immediately before a meal may
tire the client and impair the ability to eat.
Percussion and vibration, components of chest Answer B. A positive reaction means the client
physiotherapy, may worsen bronchospasms; has been exposed to TB; it isn’t conclusive of
therefore, the procedure is contraindicated in the presence of active disease. A positive
clients with bronchospasms. Secretions that reaction consists of palpable swelling and
have mobilized (especially when suction induration of 5 to 15 mm. It can be read 48 to
equipment isn’t available) are a 72 hours after the injection. In clients with
contraindication for postural drainage, another positive reactions, further studies are usually
component of chest physiotherapy. done to rule out active disease. In
immunosuppressed clients, a negative reaction
doesn’t exclude the presence of active disease.
Answer D. Hypotension, hypothermia, and
vasoconstriction may alter pulse oximetry
values by reducing arterial blood flow. Likewise, Answer A. In a client with emphysema,
movement of the finger to which the oximeter albuterol is used as a bronchodilator. A
is applied may interfere with interpretation of respiratory rate of 22 breaths/minute indicates
SaO2. All of these conditions limit the that the drug has achieved its therapeutic effect
usefulness of pulse oximetry. Fever, tachypnea, because fewer respirations are required to
and tachycardia don’t affect pulse oximetry achieve oxygenation. Albuterol has no effect on
pupil reaction or urine output. It may cause a
Reviewer

change in the heart rate, but this is an adverse, Continuous bubbling in the water-seal chamber
not therapeutic, effect. indicates a leak in the closed chest drainage
system, which must be corrected. The nurse
should keep the collection chamber below chest
Answer B. A pH less than 7.35 is indicative of
level to allow fluids to drain into it. The nurse
acidosis; a pH above 7.45 indicates alkalosis. should not strip chest tubes because doing so
may traumatize the tissue or dislodge the tube.
Answer B.
Before weaning a client from mechanical
ventilation, it’s most important to have baseline Answer B. Conditions that increase oxygen
ABG levels. During the weaning process, ABG demands include obesity, smoking, exposure to
levels will be checked to assess how the client is temperature extremes, and stress. A client with
tolerating the procedure. Other assessment chronic bronchitis should drink at least 2,000 ml
parameters are less critical. Measuring fluid of fluid daily to thin mucus secretions;
volume intake and output is always important restricting fluid intake may be harmful. The
when a client is being mechanically ventilated. nurse should encourage the client to eat a high-
Prior attempts at weaning and ECG results are
protein snack at bedtime because protein
documented on the client’s record, and the digestion produces an amino acid with sedating
nurse can refer to them before the weaning effects that may ease the insomnia associated
process begins. with chronic bronchitis. Eating more than three
large meals a day may cause fullness, making
Answer B. The ABG results reveal respiratory breathing uncomfortable and difficult; however,
alkalosis. The best intervention to raise the it doesn’t increase oxygen demands. To help
PaCO2 level would be to have the client breathe maintain adequate nutritional intake, the client
into a paper bag. All of the other options — with chronic bronchitis should eat small,
such as administering a decongestant, offering frequent meals (up to six a day).
fluids frequently, and administering
supplemental oxygen — wouldn’t raise the Answer B. Skin color doesn’t affect the mucous
lowered PaCO2 level. membranes. The lips, nail beds, and earlobes
are less reliable indicators of cyanosis because
Answer C. The most significant and direct they’re affected by skin color.
indicator of the effectiveness of oxygen therapy
is the PaO2 value. Based on the PaO2 value, the Answer D. The therapeutic serum theophylline
nurse may adjust the type of oxygen delivery concentration ranges from 10 to 20 mcg/ml.
(cannula, venturi mask, or mechanical Values below 10 mcg/ml aren’t therapeutic.
ventilator), flow rate, and oxygen percentage.
The other options reflect the client’s ventilation
status, not oxygenation. Answer A. To avoid a hypotensive reaction from
rapid I.V. administration, the nurse should
infuse vancomycin slowly, over 60 to 90
Answer A. Endotracheal suctioning removes minutes, in a large volume of fluid. Although
secretions as well as gases from the airway and neutropenia may occur in approximately 5% to
lowers the arterial oxygen saturation (SaO2) 10% of clients receiving vancomycin, this
level. Coughing and incentive spirometry adverse effect reverses rapidly when the drug is
improves oxygenation and should raise or discontinued.
maintain oxygen saturation. Because of
superficial vasoconstriction, using a cooling
blanket can lower peripheral oxygen saturation Answer C. Extreme anxiety may lead to
readings, but SaO2 levels wouldn’t be affected. respiratory alkalosis by causing
hyperventilation, which results in excessive
carbon dioxide (CO2) loss. Other conditions that
Answer A. The nurse should measure and may set the stage for respiratory alkalosis
document the amount of chest tube drainage include fever, heart failure, and injury to the
regularly to detect abnormal drainage patterns, brain’s respiratory center, overventilation with
such as may occur with a hemorrhage (if a mechanical ventilator, pulmonary embolism,
excessive) or a blockage (if decreased).
Reviewer

and early salicylate intoxication. Type 1 diabetes learn diaphragmatic (abdominal) breathing. In
mellitus may lead to diabetic ketoacidosis; the pursed-lip breathing, the client mimics a normal
deep, rapid respirations occurring in this inspiratory-expiratory (I:E) ratio of 1:2. (A client
disorder (Kussmaul’s respirations) don’t cause with emphysema may have an I:E ratio as high
excessive CO2 loss. Myasthenia gravis and as 1:4.)
narcotic overdose suppress the respiratory
drive, causing CO2 retention, not CO2 loss; this
may lead to respiratory acidosis, not alkalosis.

1. Dr. Jones prescribes albuterol sulfate


Answer D. The client is hypoxemic because of (Proventil) for a patient with newly
bronchoconstriction as evidenced by wheezes diagnose asthma. When teaching the patient
and a subnormal arterial oxygen saturation about this drug, the nurse should explain that
level. The client’s greatest need is it may cause:
bronchodilation, which can be accomplished by
administering bronchodilators. Albuterol is a A. Nasal congestion
beta2 adrenergic agonist, which causes dilation B. Nervousness
of the bronchioles. It’s given by nebulization or C. Lethargy
metered-dose inhalation and may be given as D. Hyperkalemia
often as every 30 to 60 minutes until relief is
2. Miriam, a college student with
accomplished. Alprazolam is an anxiolytic and
acute rhinitis sees the campus nurse because of
central nervous system depressant, which could
excessive nasal drainage. The nurse asks the
suppress the client’s breathing. Propranolol is
patient about the color of the drainage. In
contraindicated in a client who’s wheezing
acute rhinitis, nasal drainage normally is:
because it’s a beta2 adrenergic antagonist.
Morphine is a respiratory center depressant and A. Yellow
is contraindicated in this situation. B. Green
C. Clear
D. Gray
Answer C. The client with COPD retains carbon
dioxide, which inhibits stimulation of breathing 3. A male adult patient hospitalized for
by the medullary center in the brain. As a result, treatment of a pulmonary embolism develops
low oxygen levels in the blood stimulate respiratory alkalosis. Which clinical findings
respiration, and administering unspecified, commonly accompany respiratory alkalosis?
unmonitored amounts of oxygen may depress
ventilation. To promote adequate gas exchange, A. Nausea or vomiting
the nurse should use a Venturi mask to deliver a B. Abdominal pain or diarrhea
specified, controlled amount of oxygen C. Hallucinations or tinnitus
consistently and accurately. Drinking three D. Lightheadedness or paresthesia
glasses of fluid daily wouldn’t affect gas
4. Before administering ephedrine, Nurse Tony
exchange or be sufficient to liquefy secretions,
assesses the patient’s history. Because of
which are common in COPD. Clients with COPD
ephedrine’s central nervous system (CNS)
and respiratory distress should be placed in high
effects, it is not recommended for:
Fowler’s position and shouldn’t receive
sedatives or other drugs that may further A. Patients with an acute asthma attack
depress the respiratory center. B. Patients with narcolepsy
C. Patients under age 6
D. Elderly patients
Answer A. Pursed-lip breathing helps prevent
early airway collapse. Learning this technique 5. A female patient suffers adult respiratory
helps the client control respiration during distress syndrome as a consequence of shock.
periods of excitement, anxiety, exercise, and The patient’s condition deteriorates rapidly,
respiratory distress. To increase inspiratory and endotracheal intubation and mechanical
muscle strength and endurance, the client may ventilation are initiated. When the high-
need to learn inspiratory resistive breathing. To pressure alarm on the mechanical ventilator,
decrease accessory muscle use and thus reduce alarm sounds, the nurse starts to check for the
the work of breathing, the client may need to
Reviewer

cause. Which condition triggers the high- A. Contralateral side in a simple pneumothorax
pressure alarm? B. Affected side in a hemothorax
C. Affected side in a tension pneumothorax
A. Kinking of the ventilator tubing D. Contralateral side in hemothorax
B. A disconnected ventilator tube
C. An endotracheal cuff leak 11. After undergoing a left pneumonectomy, a
D. A change in the oxygen concentration female patient has a chest tube in place for
without resetting the oxygen level alarm drainage. When caring for this patient, the
nurse must:
6. A male adult patient on mechanical
ventilation is receiving pancuronium bromide A. Monitor fluctuations in the water-seal
(Pavulon), 0.01 mg/kg I.V. as needed. Which chamber
assessment finding indicates that the patient B. Clamp the chest tube once every shift
needs another pancuronium dose? C. Encourage coughing and deep breathing
D. Milk the chest tube every 2 hours
A. Leg movement
B. Finger movement 12. When caring for a male patient who has
C. Lip movement just had a total laryngectomy, the nurse should
D. Fighting the ventilator plan to:

7. On auscultation, which finding suggests a A. Encourage oral feeding as soon as possible


right pneumothorax? B. Develop an alternative communication
method
A. Bilateral inspiratory and expiratory crackles C. Keep the tracheostomy cuff fully inflated
B. Absence of breaths sound in the right thorax D. Keep the patient flat in bed
C. Inspiratory wheezes in the right thorax
D. Bilateral pleural friction rub. 13. A male patient has a sucking stab wound to
the chest. Which action should the nurse take
8. Rhea, confused and short breath, is brought first?
to the emergency department by a family
member. The medical history reveals chronic A. Drawing blood for a hematocrit
bronchitis and hypertension. To learn more and hemoglobin level
about the current respiratory problem, the B. Applying a dressing over the wound and
doctor orders a chest x-ray and taping it on three sides
arterial blood gas (ABG) analysis. When C. Preparing a chest tube insertion tray
reviewing the ABG report, the nurses sees D. Preparing to start an I.V. line
many abbreviations. What does a lowercase
“a” in ABG value present? 14. For a patient with advanced chronic
obstructive pulmonary disease (COPD), which
A. Acid-base balance nursing action best promotes adequate gas
B. Arterial Blood exchange?
C. Arterial oxygen saturation
D. Alveoli A. Encouraging the patient to drink three
glasses of fluid daily
9. A male patient is admitted to the healthcare B. Keeping the patient in semi-Fowler’s position
facility for treatment of chronic obstructive C. Using a high-flow venture mask to deliver
pulmonary disease. Which nursing diagnosis is oxygen as prescribe
most important for this patient? D. Administering a sedative, as prescribe

A. Activity intolerance related to fatigue 15. A male patient’s X-ray result reveals
B. Anxiety related to actual threat to health bilateral white-outs, indicating adult
status respiratory distress syndrome (ARDS). This
C. Risk for infection related to retained secretions syndrome results from:
D. Impaired gas exchange related to airflow
A. Cardiogenic pulmonary edema
obstruction
B. Respiratory alkalosis
10. Nurse Ruth assessing a patient for tracheal C. Increased pulmonary capillary permeability
displacement should know that the trachea will D. Renal failure
deviate toward the:
Reviewer

16. For a female patient with chronic 21. While changing the tapes on
obstructive pulmonary disease, which nursing a tracheostomy tube, the male client coughs
intervention would help maintain a patent and tube is dislodged. The initial nursing action
airway? is to:

A. Restricting fluid intake to 1,000 ml per day A. Call the physician to reinsert the tube
B. Enforcing absolute bed rest B. Grasp the retention sutures to spread the
C. Teaching the patient how to perform opening
controlled coughing C. Call the respiratory therapy department to
D. Administering prescribe sedatives regularly reinsert the tracheotomy
and in large amounts D. Cover the tracheostomy site with a sterile
dressing to prevent infection
17. Nurse Lei caring for a client with a
pneumothorax and who has had a chest tube 22. Nurse Oliver is caring for a client
inserted notes continues gentle bubbling in the immediately after removal of the endotracheal
suction control chamber. What action is tube. The nurse reports which of the following
appropriate? signs immediately if experienced by the client?

A. Do nothing, because this is an expected A. Stridor


finding B. Occasional pink-tinged sputum
B. Immediately clamp the chest tube and notify C. A few basilar lung crackles on the right
the physician D. Respiratory rate 24 breaths/min
C. Check for an air leak because the bubbling
23. An emergency room nurse is assessing a
should be intermittent
D. Increase the suction pressure so that the male client who has sustained a blunt injury to
bubbling becomes vigorous the chest wall. Which of these signs would
indicate the presence of a pneumothorax in
18. Nurse Maureen has assisted a physician this client?
with the insertion of a chest tube. The nurse
monitors the client and notes fluctuation of A. A low respiratory rate
the fluid level in the water seal chamber after B. Diminished breath sounds
the tube is inserted. Based on this assessment, C. The presence of a barrel chest
which action would be appropriate? D. A sucking sound at the site of injury

A. Inform the physician 24. Nurse Reese is caring for a client


B. Continue to monitor the client hospitalized with acute exacerbation of chronic
C. Reinforce the occlusive dressing obstructive pulmonary disease. Which of the
following would the nurse expect to note on
D. Encourage the client to deep breathe
assessment of this client?
19. Nurse Reynolds caring for a client with a
chest tube turns the client to the side, and the A. Hypocapnia
chest tube accidentally disconnects. The initial B. A hyperinflated chest noted on the chest x-
ray
nursing action is to:
C. Increased oxygen saturation with exercise
A. Call the physician D. A widened diaphragm noted on the chest x-
B. Place the tube in bottle of sterile water ray
C. Immediately replace the chest tube system
D. Place a sterile dressing over the 25. An oxygen delivery system is prescribed for
disconnection site a male client with chronic obstructive pulmonary
disease to deliver a precise oxygen
20. A nurse is assisting a physician with the concentration. Which of the following types of
removal of a chest tube. The nurse should oxygen delivery systems would the nurse
instruct the client to: anticipate to be prescribed?

A. Exhale slowly A. Face tent


B. Stay very still B. Venturi mask
C. Inhale and exhale quickly C. Aerosol mask
D. Perform the Valsalva maneuver D. Tracheostomy collar
Reviewer

26. Blessy, a community health nurse is A. Limiting fluid


conducting an educational session with B. Having the client take deep breaths
community members regarding tuberculosis. C. Asking the client to spit into the collection
The nurse tells the group that one of the first container
symptoms associated with tuberculosis is: D. Asking the client to obtain the specimen after
eating
A. Dyspnea
B. Chest pain 32. Nurse Joy is caring for a client after a
C. A bloody, productive cough bronchoscopy and biopsy. Which of the
D. A cough with the expectoration of mucoid following signs, if noticed in the client, should
sputum be reported immediately to the physician?

27. A nurse performs an admission assessment A. Dry cough


on a female client with a diagnosis of B. Hematuria
tuberculosis. The nurse reviews the result of C. Bronchospasm
which diagnosis test that will confirm this D. Blood-streaked sputum
diagnosis?
33. A nurse is suctioning fluids from a male
A. Bronchoscopy client via a tracheostomy tube. When
B. Sputum culture suctioning, the nurse must limit the suctioning
C. Chest x-ray time to a maximum of:
D. Tuberculin skin test
A. 1 minute
28. A nurse is caring for a male client B. 5 seconds
with emphysema who is receiving oxygen. The C. 10 seconds
nurse assesses the oxygen flow rate to ensure D. 30 seconds
that it does not exceed:
34. A nurse is suctioning fluids from a female
A. 1 L/min client through an endotracheal tube. During
B. 2 L/min the suctioning procedure, the nurse notes on
C. 6 L/min the monitor that the heart rate is decreasing.
D. 10 L/min Which if the following is the appropriate
nursing intervention?
29. A nurse instructs a female client to use the
pursed-lip method of breathing and the client A. Continue to suction
asks the nurse about the purpose of this type B. Notify the physician immediately
of breathing. The nurse responds, knowing C. Stop the procedure and reoxygenate the
that the primary purpose of pursed-lip client
breathing is to: D. Ensure that the suction is limited to 15
seconds
A. Promote oxygen intake
B. Strengthen the diaphragm 35. A male adult client is suspected of having a
C. Strengthen the intercostal muscles pulmonary embolus. A nurse assesses the
D. Promote carbon dioxide elimination client, knowing that which of the following is a
common clinical manifestation of pulmonary
30. A nurse is caring for a male client with embolism?
acute respiratory distress syndrome. Which of
the following would the nurse expect to note A. Dyspnea
in the client? B. Bradypnea
C. Bradycardia
A. Pallor D. Decreased respirations
B. Low arterial PaO2
C. Elevated arterial PaO2 36. A slightly obese female client with a history
D. Decreased respiratory rate of allergy-induced asthma, hypertension, and
mitral valve prolapse is admitted to an acute
31. A nurse is preparing to obtain a sputum care facility for elective surgery. The nurse
specimen from a male client. Which of the obtains a complete history and performs a
following nursing actions will facilitate thorough physical examination, paying special
obtaining the specimen? attention to the cardiovascular and respiratory
Reviewer

systems. When percussing the client’s chest C. Decreased hearing acuity


wall, the nurse expects to elicit: D. Increased appetite

A. Resonant sounds. 42. A male client is asking the nurse a question


B. Hyperresonant sounds. regarding the Mantoux test for tuberculosis.
C. Dull sounds. The nurse should base her response on the fact
D. Flat sounds. that the:

37. A male client who weighs 175 lb (79.4 kg) is A. Area of redness is measured in 3 days and
receiving aminophylline (Aminophyllin) (400 determines whether tuberculosis is present.
mg in 500 ml) at 50 ml/hour. The theophylline B. Skin test doesn’t differentiate between active
level is reported as 6 mcg/ml. The nurse calls and dormant tuberculosis infection.
the physician who instructs the nurse to C. Presence of a wheal at the injection site in 2
change the dosage to 0.45 mg/kg/hour. The days indicates active tuberculosis.
nurse should: D. Test stimulates a reddened response in some
clients and requires a second test in 3 months.
A. Question the order because it’s too low.
B. Question the order because it’s too high. 43. A female adult client has a tracheostomy
C. Set the pump at 45 ml/hour. but doesn’t require continuous mechanical
D. Stop the infusion and have the laboratory ventilation. When weaning the client from the
repeat the theophylline measurement. tracheostomy tube, the nurse initially should
plug the opening in the tube for:
38. The nurse is teaching a male client with
chronic bronchitis about breathing exercises. A. 15 to 60 seconds.
Which of the following should the nurse B. 5 to 20 minutes.
include in the teaching? C. 30 to 40 minutes.
D. 45 to 60 minutes.
A. Make inhalation longer than exhalation.
B. Exhale through an open mouth. 44. Nurse Oliver observes constant bubbling in
C. Use diaphragmatic breathing. the water-seal chamber of a closed chest
D. Use chest breathing. drainage system. What should the nurse
conclude?
39. Which phrase is used to describe the
volume of air inspired and expired with a A. The system is functioning normally
normal breath? B. The client has a pneumothorax.
C. The system has an air leak.
A. Total lung capacity D. The chest tube is obstructed.
B. Forced vital capacity
C. Tidal volume 45. A black client with asthma seeks
D. Residual volume emergency care for acute respiratory distress.
Because of this client’s dark skin, the nurse
40. A male client abruptly sits up in bed,
should assess for cyanosis by inspecting the:
reports having difficulty breathing and has an
arterial oxygen saturation of 88%. Which mode A. Lips.
of oxygen delivery would most likely reverse B. Mucous membranes.
the manifestations? C. Nail beds.
D. Earlobes.
A. Simple mask
B. Non-rebreather mask 46. For a male client with an endotracheal (ET)
C. Face tent tube, which nursing action is most essential?
D. Nasal cannula
A. Auscultating the lungs for bilateral breath
41. A female client must take streptomycin for sounds
tuberculosis. Before therapy begins, the nurse B. Turning the client from side to side every 2
should instruct the client to notify the hours
physician if which health concern occurs? C. Monitoring serial blood gas values every 4
hours
A. Impaired color discrimination D. Providing frequent oral hygiene
B. Increased urinary frequency
Reviewer

47. The nurse assesses a male client’s D. It prolongs the inspiratory phase of
respiratory status. Which observation indicates respiration.
that the client is experiencing difficulty
breathing? 52. After receiving an oral dose of codeine for
an intractable cough, the male client asks the
A. Diaphragmatic breathing nurse, “How long will it take for this drug to
B. Use of accessory muscles work?” How should the nurse respond?
C. Pursed-lip breathing
A. In 30 minutes
D. Controlled breathing
B. In 1 hour
48. A female client is undergoing a complete C. In 2.5 hours
physical examination as a requirement for D. In 4 hours
college. When checking the client’s respiratory
status, the nurse observes respiratory 53. A male client suffers adult respiratory
excursion to help assess: distress syndrome as a consequence of shock.
The client’s condition deteriorates rapidly, and
A. Lung vibrations. endotracheal (ET) intubation and mechanical
B. Vocal sounds. ventilation are initiated. When the high-
C. Breath sounds. pressure alarm on the mechanical ventilator
D. Chest movements. sounds, the nurse starts to check for the cause.
Which condition triggers the high-pressure
49. A male client comes to the emergency alarm?
department complaining of sudden onset
of diarrhea, anorexia, malaise, cough, A. Kinking of the ventilator tubing
headache, and recurrent chills. Based on the B. A disconnected ventilator tube
client’s history and physical findings, the C. An ET cuff leak
physician suspects legionnaires’ disease. While D. A change in the oxygen concentration
awaiting diagnostic test results, the client is without resetting the oxygen level alarm
admitted to the facility and started
on antibiotic therapy. What is the drug of 54. A female client with chronic obstructive
choice for treating legionnaires’ disease? pulmonary disease (COPD) takes anhydrous
theophylline, 200 mg P.O. every 8 hours.
A. Erythromycin (Erythrocin) During a routine clinic visit, the client asks the
B. Rifampin (Rifadin) nurse how the drug works. What is the
C. Amantadine (Symmetrel) mechanism of action of anhydrous
D. Amphotericin B (Fungizone) theophylline in treating a nonreversible
obstructive airway disease such as COPD?
50. A male client with chronic obstructive
pulmonary disease (COPD) is recovering from A. It makes the central respiratory center more
a myocardial infarction. Because the client is sensitive to carbon dioxide and stimulates the
extremely weak and can’t produce an effective respiratory drive.
cough, the nurse should monitor closely for: B. It inhibits the enzyme phosphodiesterase,
decreasing degradation of cyclic adenosine
A. Pleural effusion. monophosphate, a bronchodilator.
B. Pulmonary edema. C. It stimulates adenosine receptors, causing
C. Atelectasis.
bronchodilation.
D. Oxygen toxicity. D. It alters diaphragm movement, increasing
51. The nurse in charge is teaching a client with chest expansion and enhancing the lung’s
emphysema how to perform pursed-lip capacity for gas exchange.
breathing. The client asks the nurse to explain 55. A male client with
the purpose of this breathing technique. Which pneumococcal pneumonia is admitted to an
explanation should the nurse provide? acute care facility. The client in the next room
A. It helps prevent early airway collapse. is being treated for mycoplasmal pneumonia.
B. It increases inspiratory muscle strength. Despite the different causes of the various
C. It decreases use of accessory breathing types of pneumonia, all of them share which
muscles. feature?
Reviewer

A. Inflamed lung tissue 60. The nurse in charge formulates a nursing


B. Sudden onset diagnosis of Activity intolerance related to
C. Responsiveness to penicillin. inadequate oxygenation and dyspnea for a
D. Elevated white blood cell (WBC) count client with chronic bronchitis. To minimize this
problem, the nurse instructs the client to avoid
56. A client with Guillain-Barré conditions that increase oxygen demands.
syndrome develops respiratory acidosis as a
Such conditions include:
result of reduced alveolar ventilation. Which
combination of arterial blood gas (ABG) values A. Drinking more than 1,500 ml of fluid daily.
confirms respiratory acidosis? B. Being overweight.
C. Eating a high-protein snack at bedtime.
A. pH, 5.0; PaCO2 30 mm Hg D. Eating more than three large meals a day.
B. pH, 7.40; PaCO2 35 mm Hg
C. pH, 7.35; PaCO2 40 mm Hg Answers and Rationale
D. pH, 7.25; PaCO2 50 mm Hg
1. Answer: B. Nervousness
57. A male client admitted to an acute care
facility with pneumonia is receiving Albuterol may cause nervousness. The inhaled
supplemental oxygen, 2 L/minute via nasal form of the drug may cause dryness and
cannula. The client’s history includes chronic irritation of the nose and throat, not nasal
obstructive pulmonary disease (COPD) congestion; insomnia, not lethargy;
and coronary artery disease. Because of these and hypokalemia (with high doses),
history findings, the nurse closely monitors the not hyperkalemia. Other adverse effects of
albuterol include tremor, dizziness, headache,
oxygen flow and the client’s respiratory status.
Which complication may arise if the client tachycardia, palpitations, hypertension,
receives a high oxygen concentration? heartburn, nausea, vomiting and muscle
cramps.
A. Apnea
B. Anginal pain 2. Answer: C. Clear
C. Respiratory alkalosis Normally, nasal drainage in acute rhinitis is
D. Metabolic acidosis clear. Yellow or green drainage indicates spread
58. At 11 p.m., a male client is admitted to the of the infection to the sinuses. Gray drainage
emergency department. He has a respiratory may indicate a secondary infection.
rate of 44 breaths/minute. He’s anxious, and 3. Answer: D. Lightheadedness or paresthesia
wheezes are audible. The client is immediately
given oxygen by face mask The patient with respiratory alkalosis may
and methylprednisolone (Depo-medrol) I.V. At complain of lightheadedness or paresthesia
11:30 p.m., the client’s arterial blood oxygen (numbness and tingling in the arms and legs).
saturation is 86% and he’s still wheezing. The Nausea, vomiting, abdominal pain, and diarrhea
nurse should plan to administer: may accompany respiratory acidosis.
Hallucinations and tinnitus rare are associated
A. Alprazolam (Xanax). with respiratory alkalosis or any other acid-base
B. Propranolol (Inderal) imbalance.
C. Morphine.
D. Albuterol (Proventil). 4. Answer: D. Elderly patients

59. After undergoing a thoracotomy, a male Ephedrine is not recommended for elderly
client is receiving epidural analgesia. Which patients, who are particularly susceptible to
assessment finding indicates that the client has CNS reactions (such as confusion and anxiety)
developed the most serious complication of and to cardiovascular reactions (such as
epidural analgesia? increased systolic blood pressure, coldness in the
extremities, and anginal pain). Ephedrine is
A. Heightened alertness used for its bronchodilator effects with acute
B. Increased heart rate and chronic asthma and occasionally for its CNS
C. Numbness and tingling of the extremities stimulant actions for narcolepsy. It can be
D. Respiratory depression administered to children age 2 and older.
Reviewer

5. Answer: A. Kinking of the ventilator tubing making “impaired gas exchange related to
airflow obstruction” the most important nursing
Conditions that trigger the high-pressure alarm diagnosis. The other options also may apply to
include kinking of the ventilator tubing, this patient but less important.
bronchospasm or pulmonary embolus, mucus
plugging, water in the tube, coughing or biting 10. Answer: D. Contralateral side in
on endotracheal tube, and the patient’s being hemothorax
out of breathing rhythm with the ventilator. A
disconnected ventilator tube or an The trachea will shift according to the pressure
endotracheal cuff leak would trigger the low gradients within the thoracic cavity. In tension
pressure alarm. Changing the oxygen pneumothorax and hemothorax, accumulation
concentration without resetting the oxygen of air or fluid causes a shift away from the
injured side. If there is no significant air or fluid
level alarm would trigger the oxygen alarm.
accumulation, the trachea will not shift.
6. Answer: D. Fighting the ventilator Tracheal deviation toward the contralateral side
in simple pneumothorax is seen when the
Pancuronium, a nondepolarizing blocking agent, thoracic contents shift in response to the
is used for muscle relaxation and paralysis. It release of normal thoracic pressure gradients
assists mechanical ventilation by promoting
on the injured side.
endotracheal intubation and paralyzing the
patient so that the mechanical ventilator can do 11. Answer: C. Encourage coughing and deep
its work. Fighting the ventilator is a sign that the breathing
patient needs another pancuronium dose. The
When caring for a patient who is recovering
nurse should administer 0.01 to 0.02 mg/kg I.V.
every 20 to 60 minutes. Movement of the legs, from a pneumonectomy, the nurse should
or lips has no effect on the ventilator and encourage coughing and deep breathing to
therefore is not used to determine the need for prevent pneumonia in the unaffected lung.
Because the lung has been removed, the water-
another dose.
seal chamber should display no fluctuations.
7. Answer: B. Absence of breaths sound in the Reinflation is not the purpose of chest tube.
right thorax Chest tube milking is controversial and should
be done only to remove blood clots that
In pneumothorax, the alveoli are deflated and obstruct the flow of drainage.
no air exchange occurs in the lungs. Therefore,
breath sounds in the affected lung field are 12. Answer: B. Develop an alternative
absent. None of the other options are communication method
associated with pneumothorax. Bilateral
crackles may result from pulmonary congestion, A patient with a laryngectomy cannot speak, yet
inspiratory wheezes may signal asthma, and a still needs to communicate. Therefore, the
pleural friction rub may indicate pleural nurse should plan to develop an alternative
communication method. After a laryngectomy,
inflammation.
edema interferes with the ability to swallow
8. Answer: B. Arterial Blood and necessitates tube (enteral) feedings. To
prevent injury to the tracheal mucosa, the nurse
A lowercase “a” in an ABG value represents should deflate the tracheostomy cuff or use the
arterial blood. For instance, the abbreviation
minimal leak technique. To decrease edema,
PaO2 refers to the partial pressure of oxygen in the nurse should place the patient in semi-
arterial blood. The pH value reflects the acid- Fowler’s position.
base balance in arterial blood. Sa02 indicates
arterial oxygen saturation. An uppercase “A” 13. Answer: B. Applying a dressing over the
represents alveolar conditions: for example, wound and taping it on three sides
PA02 indicates the partial pressure of oxygen in
the alveoli. The nurse immediately should apply a dressing
over the stab wound and tape it on three sides
9. Answer: D. Impaired gas exchange related to to allow air to escape and to prevent tension
airflow obstruction pneumothorax (which is more life-threatening
than an open chest wound). Only after covering
A patient airway and an adequate breathing and taping the wound should the nurse draw
pattern are the top priority for any patient,
Reviewer

blood for laboratory tests, assist with chest tube continuous and not intermittent. Option d is
insertion, and start an I.V. line. incorrect because bubbling should be gentle.
Increasing the suction pressure only increases
14. Answer: C. Using a high-flow venture mask the rate of evaporation of water in the drainage
to deliver oxygen as prescribe system.
The patient with COPD retains carbon dioxide,
18. Answer: B. Continue to monitor the client
which inhibits stimulation of breathing by the
medullary center in the brain. As a result, low The presence of fluctuation of the fluid level in
oxygen levels in the blood stimulate respiration, the water seal chamber indicates a patent
and administering unspecified, unmonitored drainage system. With normal breathing, the
amounts of oxygen may depress ventilation. To water level rises with inspiration and falls
promote adequate gas exchange, the nurse with expiration. Fluctuation stops if the tube is
should use a Venturi mask to deliver a specified, obstructed, if a dependent loop exists, if the
controlled amount of oxygen consistently and suction is not working properly, or if the lung
accurately. Drinking three glasses of fluid daily has reexpanded. Options A, C, and D are
would not affect gas exchange or be sufficient incorrect.
to liquefy secretions, which are common in
COPD. Patients with COPD and respiratory 19. Answer: B. Place the tube in bottle of
distress should be places in high-Fowler’s sterile water
position and should not receive sedatives or If the chest drainage system is disconnected,
other drugs that may further depress the the end of the tube is placed in a bottle of
respiratory center. sterile water held below the level of the chest.
15. Answer: C. Increased pulmonary capillary The system is replaced if it breaks or cracks or if
permeability the collection chamber is full. Placing a sterile
dressing over the disconnection site will not
ARDS results from increased pulmonary prevent complications resulting from the
capillary permeability, which leads to disconnection. The physician may need to be
noncardiogenic pulmonary edema. notified, but this is not the initial action.
In cardiogenic pulmonary edema, pulmonary
congestion occurs secondary to heart failure. In 20. Answer: D. Perform the Valsalva maneuver
the initial stage of ARDS, respiratory alkalosis When the chest tube is removed, the client is
may arise secondary to hyperventilation; asked to perform the Valsalva maneuver (take a
however, it does not cause ARDS. Renal failure deep breath, exhale, and bear down). The tube
does not cause ARDS, either. is quickly withdrawn, and an airtight dressing is
16. Answer: C. Teaching the patient how to taped in place. An alternative instruction is to
ask the client to take a deep breath and hold
perform controlled coughing
the breath while the tube is removed. Options
Controlled coughing helps maintain a patent A, B, and C are incorrect client instructions.
airway by helping to mobilize and remove
secretions. A moderate fluid intake (usually 2 L 21. Answer: B. Grasp the retention sutures to
or more daily) and moderate activity help spread the opening
liquefy and mobilize secretions. Bed rest and If the tube is dislodged accidentally, the initial
sedatives may limit the patient’s ability to nursing action is to grasp the retention sutures
maintain a patent airway, causing a high risk for and spread the opening. If agency policy
infection from pooled secretions. permits, the nurse then attempts immediately
17. Answer: A. Do nothing, because this is an to replace the tube. Covering the tracheostomy
site will block the airway. Options A and C will
expected finding
delay treatment in this emergency situation.
Continuous gentle bubbling should be noted in
the suction control chamber. Option b is 22. Answer: A. Stridor
incorrect. Chest tubes should only be clamped The nurse reports stridor to the physician
to check for an air leak or when changing immediately. This is a high-pitched, coarse
drainage devices (according to agency policy). sound that is heard with the stethoscope over
Option c is incorrect. Bubbling should be the trachea. Stridor indicates airway edema and
Reviewer

places the client at risk for airway obstruction. Oxygen is used cautiously and should not
Options B, C, and D are not signs that require exceed 2 L/min. Because of the long-standing
immediate notification of the physician. hypercapnia that occurs in emphysema, the
respiratory drive is triggered by low oxygen
23. Answer: B. Diminished breath sounds levels rather than increased carbon dioxide
This client has sustained a blunt or a closed levels, as is the case in a normal respiratory
chest injury. Basic symptoms of a closed system.
pneumothorax are shortness of breath and 29. Answer: D. Promote carbon dioxide
chest pain. A larger pneumothorax may cause
elimination
tachypnea, cyanosis, diminished breath sounds,
and subcutaneous emphysema. Pursed-lip breathing facilitates maximal
Hyperresonance also may occur on the affected expiration for clients with obstructive lung
side. A sucking sound at the site of injury would disease. This type of breathing allows better
be noted with an open chest injury. expiration by increasing airway pressure that
keeps air passages open during exhalation.
24. Answer: B. A hyperinflated chest noted on Options A, B, and C are not the purposes of this
the chest x-ray type of breathing.
Clinical manifestations of chronic obstructive 30. Answer: B. Low arterial PaO2
pulmonary disease (COPD) include hypoxemia,
hypercapnia, dyspnea on exertion and at rest, The earliest clinical sign of acute respiratory
oxygen desaturation with exercise, and the use distress syndrome is an increased respiratory
of accessory muscles of respiration. Chest x-rays rate. Breathing becomes labored, and the client
reveal a hyperinflated chest and a flattened may exhibit air hunger, retractions, and
diaphragm if the disease is advanced. cyanosis. Arterial blood gas analysis reveals
increasing hypoxemia, with a PaO2 lower than
25. Answer: B. Venturi mask 60 mm Hg.
The Venturi mask delivers the most accurate 31. Answer: B. Having the client take deep
oxygen concentration. It is the best oxygen
breaths
delivery system for the client with chronic
airflow limitation because it delivers a precise To obtain a sputum specimen, the client should
oxygen concentration. The face tent, aerosol rinse the mouth to reduce contamination,
mask, and tracheostomy collar are also high- breathe deeply, and then cough into a sputum
flow oxygen delivery systems but most often specimen container. The client should be
are used to administer high humidity. encouraged to cough and not spit so as to
obtain sputum. Sputum can be thinned by fluids
26. Answer: D. A cough with the expectoration or by a respiratory treatment such as inhalation
of mucoid sputum of nebulized saline or water. The optimal time
One of the first pulmonary symptoms is a slight to obtain a specimen is on arising in the
cough with the expectoration of mucoid morning.
sputum. Options A, B, and C are late symptoms 32. Answer: C. Bronchospasm
and signify cavitation and extensive lung
involvement. If a biopsy was performed during a
bronchoscopy, blood-streaked sputum is
27. Answer: B. Sputum culture expected for several hours. Frank blood
Tuberculosis is definitively diagnosed through indicates hemorrhage. A dry cough may be
culture and isolation of Mycobacterium expected. The client should be assessed for
tuberculosis. A presumptive diagnosis is made signs of complications, which would include
based on a tuberculin skin test, a sputum smear cyanosis, dyspnea, stridor, bronchospasm,
that is positive for acid-fast bacteria, a chest x- hemoptysis, hypotension, tachycardia, and
ray, and histological evidence of granulomatous dysrhythmias. Hematuria is unrelated to this
disease on biopsy. procedure.

28. Answer: B. 2 L/min 33. Answer: C. 10 seconds


Reviewer

Hypoxemia can be caused by prolonged strengthen the diaphragm and maximizes


suctioning, which stimulates the pacemaker cells ventilation. Exhalation should be longer than
in the heart. A vasovagal response may occur, inhalation to prevent collapse of the bronchioles.
causing bradycardia. The nurse must The client with chronic bronchitis should exhale
preoxygenate the client before suctioning and through pursed lips to prolong exhalation, keep
limit the suctioning pass to 10 seconds. the bronchioles from collapsing, and prevent air
trapping. Diaphragmatic breathing — not chest
34. Answer: C. Stop the procedure and
breathing — increases lung expansion.
reoxygenate the client
39. Answer: C. Tidal volume
During suctioning, the nurse should monitor the
client closely for side effects, including Tidal volume refers to the volume of air inspired
hypoxemia, cardiac irregularities such as a and expired with a normal breath. Total lung
decrease in heart rate resulting from vagal capacity is the maximal amount of air the lungs
stimulation, mucosal trauma, hypotension, and and respiratory passages can hold after a forced
paroxysmal coughing. If side effects develop, inspiration. Forced vital capacity is the vital
especially cardiac irregularities, the procedure is capacity performed with a maximally forced
stopped and the client is reoxygenated. expiration. Residual volume is the maximal
amount of air left in the lung after a maximal
35. Answer: A. Dyspnea expiration.
The common clinical manifestations of 40. Answer: B. Non-rebreather mask
pulmonary embolism are tachypnea,
tachycardia, dyspnea, and chest pain. A non-rebreather mask can deliver levels of the
fraction of inspired oxygen (FIO2) as high as
36. Answer: A. Resonant sounds. 100%. Other modes — simple mask, face tent,
When percussing the chest wall, the nurse and nasal cannula — deliver lower levels of
expects to elicit resonant sounds — low- FIO2.
pitched, hollow sounds heard over normal lung 41. Answer: C. Decreased hearing acuity
tissue. Hyperresonant sounds indicate increased
air in the lungs or pleural space; they’re louder Decreased hearing acuity indicates ototoxicity, a
and lower pitched than resonant sounds. serious adverse effect of streptomycin therapy.
Although hyperresonant sounds occur in such The client should notify the physician
disorders as emphysema and pneumothorax, immediately if it occurs so that streptomycin
they may be normal in children and very thin can be discontinued and an alternative drug can
adults. Dull sounds, normally heard only over be prescribed. The other options aren’t
the liver and heart, may occur over dense lung associated with streptomycin. Impaired color
tissue, such as from consolidation or a tumor. discrimination indicates color blindness;
Dull sounds are thudlike and of medium pitch. increased urinary frequency and increased
Flat sounds, soft and high-pitched, are heard appetite accompany diabetes mellitus.
over airless tissue and can be replicated by
42. Answer: B. Skin test doesn’t differentiate
percussing the thigh or a bony structure.
between active and dormant tuberculosis
37. Answer: A. Question the order because it’s infection.
too low.
The Mantoux test doesn’t differentiate between
A therapeutic theophylline level is 10 to 20 active and dormant infections. If a positive
mcg/ml. The client is currently receiving 0.5 reaction occurs, a sputum smear and culture as
mg/kg/hour of aminophylline. Because the well as a chest X-ray are necessary to provide
client’s theophylline level is sub-therapeutic, more information. Although the area of redness
reducing the dose (which is what the physician’s is measured in 3 days, a second test may be
order would do) would be inappropriate. needed; neither test indicates that tuberculosis
Therefore, the nurse should question the order. is active. In the Mantoux test, an induration 5 to
9 mm in diameter indicates a borderline
38. Answer: C. Use diaphragmatic breathing. reaction; a larger induration indicates a positive
In chronic bronchitis the diaphragm is flat and reaction. The presence of a wheal within 2 days
weak. Diaphragmatic breathing helps to doesn’t indicate active tuberculosis.
Reviewer

43. Answer: B. B. 5 to 20 minutes. thoracic expansion is symmetrical; unequal


expansion may indicate pleural effusion,
Initially, the nurse should plug the opening in atelectasis, pulmonary embolus, or a rib
the tracheostomy tube for 5 to 20 minutes, or sternum fracture. The nurse assesses vocal
then gradually lengthen this interval according
sounds to evaluate air flow when checking for
to the client’s respiratory status. A client who tactile fremitus; after asking the client to say
doesn’t require continuous mechanical “99,” the nurse palpates the vibrations
ventilation already is breathing without transmitted from the bronchopulmonary
assistance, at least for short periods; therefore, system along the solid surfaces of the chest wall
plugging the opening of the tube for only 15 to to the nurse’s palms. The nurse assesses breath
60 seconds wouldn’t be long enough to reveal
sounds during auscultation.
the client’s true tolerance to the procedure.
Plugging the opening for more than 20 minutes 49. Answer: A. Erythromycin (Erythrocin)
would increase the risk of acute respiratory
distress because the client requires an Erythromycin is the drug of choice for treating
adjustment period to start breathing normally. legionnaires’ disease. Rifampin may be added to
the regimen if erythromycin alone is ineffective;
44. Answer: C. The system has an air leak. however, it isn’t administered first. Amantadine,
an antiviral agent, and amphotericin B,
Constant bubbling in the chamber indicates an an antifungal agent, are ineffective against
air leak and requires immediate intervention. legionnaires’ disease, which is caused by
The client with a pneumothorax will have
bacterial infection.
intermittent bubbling in the water-seal
chamber. Clients without a pneumothorax 50. Answer: C. Atelectasis.
should have no evidence of bubbling in the
chamber. If the tube is obstructed, the nurse In a client with COPD, an ineffective cough
should notice that the fluid has stopped impedes secretion removal. This, in turn, causes
mucus plugging, which leads to localized airway
fluctuating in the water-seal chamber.
obstruction — a known cause of atelectasis. An
45. Answer: B. Mucous membranes. ineffective cough doesn’t cause pleural effusion
(fluid accumulation in the pleural space).
Skin color doesn’t affect the mucous Pulmonary edema usually results from left-
membranes. The lips, nail beds, and earlobes sided heart failure, not an ineffective cough.
are less reliable indicators of cyanosis because Although many noncardiac conditions may
they’re affected by skin color. cause pulmonary edema, an ineffective cough
46. Answer: A. Auscultating the lungs for isn’t one of them. Oxygen toxicity results from
bilateral breath sounds prolonged administration of high oxygen
concentrations, not an ineffective cough.
For a client with an ET tube, the most important
nursing action is auscultating the lungs regularly 51. Answer: A. It helps prevent early airway
for bilateral breath sounds to ensure proper collapse.
tube placement and effective oxygen delivery. Pursed-lip breathing helps prevent early airway
Although the other options are appropriate for collapse. Learning this technique helps the
this client, they’re secondary to ensuring client control respiration during periods of
adequate oxygenation. excitement, anxiety, exercise, and respiratory
47. Answer: B. Use of accessory muscles distress. To increase inspiratory muscle strength
and endurance, the client may need to learn
The use of accessory muscles for respiration inspiratory resistive breathing. To decrease
indicates the client is having difficulty breathing. accessory muscle use and thus reduce the work
Diaphragmatic and pursed-lip breathing are two of breathing, the client may need to learn
controlled breathing techniques that help the diaphragmatic (abdominal) breathing. In
client conserve energy. pursed-lip breathing, the client mimics a normal
inspiratory-expiratory (I:E) ratio of 1:2. (A client
48. Answer: D. Chest movements.
with emphysema may have an I:E ratio as high
The nurse observes respiratory excursion to as 1:4.)
help assess chest movements. Normally,
52. Answer: A. In 30 minutes
Reviewer

Codeine’s onset of action is 30 minutes. Its peak In respiratory acidosis, ABG analysis reveals an
concentration occurs in about 1 hour; its half- arterial pH below 7.35 and partial pressure of
life, in 2.5 hours; and its duration of action is 4 arterial carbon dioxide (PaCO2) above 45 mm
to 6 hours. Hg. Therefore, the combination of a pH value of
7.25 and a PaCO2 value of 50 mm Hg confirms
53. Answer: A. Kinking of the ventilator tubing respiratory acidosis. A pH value of 5.0 with a
Conditions that trigger the high-pressure alarm PaCO2 value of 30 mm Hg indicates respiratory
include kinking of the ventilator tubing, alkalosis. Options B and C represent normal
bronchospasm or pulmonary embolus, mucus ABG values, reflecting normal gas exchange in
plugging, water in the tube, coughing or biting the lungs.
on the ET tube, and the client’s being out of 57. Answer: A. Apnea
breathing rhythm with the ventilator. A
disconnected ventilator tube or an ET cuff leak Hypoxia is the main breathing stimulus for a
would trigger the low-pressure alarm. Changing client with COPD. Excessive oxygen
the oxygen concentration without resetting the administration may lead to apnea by removing
oxygen level alarm would trigger the oxygen that stimulus. Anginal pain results from a
alarm. reduced myocardial oxygen supply. A client with
COPD may have anginal pain from generalized
54. Answer: A. It makes the central respiratory vasoconstriction secondary to hypoxia;
center more sensitive to carbon dioxide and however, administering oxygen at any
stimulates the respiratory drive. concentration dilates blood vessels, easing
Anhydrous theophylline and other anginal pain. Respiratory alkalosis results from
methylxanthine agents make the central alveolar hyperventilation, not excessive oxygen
respiratory center more sensitive to CO2 and administration. In a client with COPD, high
stimulate the respiratory drive. Inhibition of oxygen concentrations decrease the ventilatory
phosphodiesterase is the drug’s mechanism of drive, leading to respiratory acidosis, not
action in treating asthma and other reversible alkalosis. High oxygen concentrations don’t
obstructive airway diseases — not COPD. cause metabolic acidosis.
Methylxanthine agents inhibit rather than
58. Answer: D. Albuterol (Proventil).
stimulate adenosine receptors. Although these
agents reduce diaphragmatic fatigue in clients The client is hypoxemic because of
with chronic bronchitis or emphysema, they bronchoconstriction as evidenced by wheezes
don’t alter diaphragm movement to increase and a subnormal arterial oxygen saturation
chest expansion and enhance gas exchange. level. The client’s greatest need is
bronchodilation, which can be accomplished by
55. Answer: A. Inflamed lung tissue administering bronchodilators. Albuterol is a
The common feature of all types of pneumonia beta2 adrenergic agonist, which causes dilation
is an inflammatory pulmonary response to the of the bronchioles. It’s given by nebulization or
offending organism or agent. Although most metered-dose inhalation and may be given as
types of pneumonia have a sudden onset, a few often as every 30 to 60 minutes until relief is
(such as anaerobic bacterial pneumonia and accomplished. Alprazolam is an anxiolytic and
mycoplasmal pneumonia) have an insidious central nervous system depressant, which could
onset. Antibiotic therapy is the primary suppress the client’s breathing. Propranolol is
treatment for most types of pneumonia; contraindicated in a client who’s wheezing
however, the antibiotic must be specific for the because it’s a beta2 adrenergic antagonist.
causative agent, which may not be responsive Morphine is a respiratory center depressant and
to penicillin. A few types of pneumonia, such as is contraindicated in this situation.
viral pneumonia, aren’t treated with antibiotics. 59. Answer: D. Respiratory depression
Although pneumonia usually causes an elevated
WBC count, some types, such as mycoplasmal Respiratory depression is the most serious
pneumonia, don’t. complication of epidural analgesia. Other
potential complications include hypotension,
56. Answer: D. pH, 7.25; PaCO2 50 mm Hg decreased sensation and movement of the
extremities, allergic reactions,
and urine retention. Typically, epidural analgesia
Reviewer

causes central nervous system depression A. Respiratory alkalosis


(indicated by drowsiness) as well as a decreased B. Respiratory acidosis
heart rate and blood pressure. C. Metabolic acidosis
D. Metabolic alkalosis
60. Answer: B. Being overweight.
4. Which of the following information
Conditions that increase oxygen demands
corresponds with a negative TB test?
include obesity, smoking, exposure to
temperature extremes, and stress. A client with A. 0-4 mm induration at 48 hours
chronic bronchitis should drink at least 2,000 ml B. 0-5 mm induration at 48 hours
of fluid daily to thin mucus secretions; C. 0-6 mm induration at 48 hours
restricting fluid intake may be harmful. The D. 0-7 mm induration at 48 hours
nurse should encourage the client to eat a high-
protein snack at bedtime because protein 5. Which of the following is the most common
digestion produces an amino acid with sedating type of lung cancer?
effects that may ease the insomnia associated A. Large cell
with chronic bronchitis. Eating more than three B. Adenocarcinoma
large meals a day may cause fullness, making C. Oat cell
breathing uncomfortable and difficult; however, D. Squamous cell
it doesn’t increase oxygen demands. To help
maintain adequate nutritional intake, the client 6. What cell type secrets surfactant?
with chronic bronchitis should eat small,
A. Plasma cell
frequent meals (up to six a day).
B. Type I alveolar cell
C. Type II alveolar cell
D. Type III alveolar cell
1 Which of the following conditions correlate
with the following information: 7. Which of the following pulmonary term
correlates with the definition: noted
i. High pH obstruction of the trachea or larynx.
ii. High HCO3
iii. High BE A. Rhonchi
iv. Neutral pCO2 B. Stridor
A. Respiratory alkalosis
C. Wheezes
B. Respiratory acidosis
D. Vesicular
C. Metabolic acidosis
D. Metabolic alkalosis 8. Normal values for pCO2 are considered:

2. Which of the following conditions A. 20-40 mm Hg


correlate with the following information: B. 25-30 mm Hg
C. 30-40 mm Hg
 High pH
D. 35-45 mm Hg
 Neutral HCO3
 Neutral BE 9. Normal values for HCO3 are considered:
 Low pCO2
A. 15-30 mEq/L
B. 20-35 mEq/L
A. Respiratory alkalosis
C. 22-26 mEq/L
B. Respiratory acidosis
D. 24-29 mEq/L
C. Metabolic acidosis
D. Metabolic alkalosis 10. Pneumoncystis carinii infections are
commonly treated with which of the following
3. Which of the following conditions correlate
medications?
with the following information:
A. Pentamidine
 Low pH
B. Allopurinol
 Low HCO3
C. Iorazepam
 Low BE
D. Chlorpropamide
 Neutral pCO2
Reviewer

11. Which of the following is not generally 18. Which of the following matches the
caused by COPD? definition: The volume of air that can be inhaled
following exhalation of tidal volume?
A. Pneumonia
B. Right sided heart failure A. Expiratory reserve volume
C. Headaches B. Inspiratory capacity
D. Cor pulmonale C. Inspiratory reserve volume
D. Vital capacity
12. Which of the following is not considered a
COPD related disease? 19. Which of the following matches the
definition: The maximum volume of air that can
A. Bronchiectasis
be exhaled after taking the deepest breath
B. Bronchial asthma possible?
C. Bronchitis
D. Bronchial hypotension A. Expiratory reserve volume
B. Inspiratory capacity
13. Which of the following pulmonary term C. Inspiratory reserve volume
correlates with the definition: bronchospasm of
D. Vital capacity
the bronchial walls?
20. The respiratory center is located in the ____
A. Wheezes and ______.
B. Rhonchi
C. Stridor A. Midbrain and pons
D. Pleural Rub B. Pons and Medulla oblongata
C. Midbrain and Medulla oblongata
14. Which of the following is considered an D. Pons and Hypothalamus
expectorant?
Answers
A. Acetylcysteine
B. Guaifenesin 1. Metabolic alkalosis
C. Theophylline 2. Respiratory alkalosis
D. Epinephrine HCL 3. Metabolic acidosis
4. 0-4 mm induration at 48 hours
15. Which of the following is considered a 5. Squamous cell
bronchodilator? 6. Type II alveolar cell
A. Acetylcysteine 7. Stridor
B. Guaifenesin 8. 35-45 mm Hg
C. Theophylline 9. 22-26 mEq/L
D. Epinephrine HCL 10. Pentamidine
11. Headaches
16. Which of the following is considered a 12. Bronchial hypotension
xanthine? 13. Wheezes
14. Guaifenesin
A. Acetylcysteine
15. Epinephrine HCL
B. Guaifenesin
16. Theophylline
C. Theophylline
17. Acetylcysteine
D. Epinephrine HCL
18. Inspiratory capacity
17. Which of the following is considered a 19. Vital capacity
mucolytic? 20. Pons and Medulla oblongata

A. Acetylcysteine
B. Guaifenesin
URINARY SYSTEM
C. Theophylline
D. Epinephrine HCL 1. Which of the following symptoms do you
expect to see in a patient diagnosed with acute
pyelonephritis?
Reviewer

1. Jaundice and flank pain 1. Insert I.V. lines above the fistula.
2. Costovertebral angle tenderness and chills 2. Avoid taking blood pressures in the arm with
3. Burning sensation on urination the fistula.
4. Polyuria and nocturia 3. Palpate pulses above the fistula.
4. Report a bruit or thrill over the fistula to the
2. You have a patient that might have a urinary doctor.
tract infection (UTI). Which statement by the
patient suggests that a UTI is likely? 8. Your patient becomes restless and tells you
she has a headache and feels nauseous during
1. “I pee a lot.” hemodialysis. Which complication do you
2. “It burns when I pee.” suspect?
3. “I go hours without the urge to pee.”
4. “My pee smells sweet.” 1. Infection
2. Disequilibrium syndrome
3. Which instructions do you include in the 3. Air embolus
teaching care plan for a patient
4. Acute hemolysis
with cystitis receiving phenazopyridine
(Pyridium). 9. Your patient is complaining
of muscle cramps while undergoing
1. If the urine turns orange-red, call the doctor. hemodialysis. Which intervention is effective
2. Take phenazopyridine just before urination to
in relieving muscle cramps?
relieve pain.
3. Once painful urination is relieved, discontinue 1. Increase the rate of dialysis
prescribed antibiotics. 2. Infuse normal saline solution
4. After painful urination is relieved, stop taking 3. Administer a 5% dextrose solution
phenazopyridine. 4. Encourage active ROM exercises

4. Which patient is at greatest risk for 10. Your patient with chronic renal failure
developing a urinary tract infection (UTI)? reports pruritus. Which instruction should you
include in this patient’s teaching plan?
1. A 35 y.o. woman with a fractured wrist
2. A 20 y.o. woman with asthma 1. Rub the skin vigorously with a towel
3. A 50 y.o. postmenopausal woman 2. Take frequent baths
4. A 28 y.o. with angina 3. Apply alcohol-based emollients to the skin
4. Keep fingernails short and clean
5. You have a patient that is receiving
peritoneal dialysis. What should you do when 11. Which intervention do you plan to include
you notice the return fluid is slowly draining? with a patient who has renal calculi?

1. Check for kinks in the outflow tubing. 1. Maintain bed rest


2. Raise the drainage bag above the level of the 2. Increase dietary purines
abdomen. 3. Restrict fluids
3. Place the patient in a 4. Strain all urine
reverse Trendelenburg position.
4. Ask the patient to cough. 12. An 18 y.o. student is admitted with dark
urine, fever, and flank pain and is diagnosed
6. What is the appropriate infusion time for with acute glomerulonephritis. Which would
the dialysate in your 38 y.o. patient with most likely be in this student’s health history?
chronic renal failure?
1. Renal calculi
1. 15 minutes 2. Renal trauma
2. 30 minutes 3. Recent sore throat
3. 1 hour 4. Family history of acute glomerulonephritis
4. 2 to 3 hours
13. Which drug is indicated for pain related to
7. A 30 y.o. female patient is undergoing acute renal calculi?
hemodialysis with an internal arteriovenous
fistula in place. What do you do to prevent 1. Narcotic analgesics
2. Nonsteroidal anti-inflammatory drugs
complications associated with this device?
(NSAIDS)
Reviewer

3. Muscle relaxants 3. Excretory urography


4. Salicylates 4. Prostate-specific antigen

14. Which of the following causes the majority 21. A 22 y.o. patient with diabetic nephropathy
of UTI’s in hospitalized patients? says, “I have two kidneys and I’m still young. If
I stick to my insulin schedule, I don’t have to
1. Lack of fluid intake worry about kidney damage, right?” Which of
2. Inadequate perineal care the following statements is the best response?
3. Invasive procedures
4. Immunosuppression 1. “You have little to worry about as long as
your kidneys keep making urine.”
15. Clinical manifestations of acute
2. “You should talk to your doctor because
glomerulonephritis include which of the statistics show that you’re being unrealistic.”
following? 3. “You would be correct if your diabetes could
1. Chills and flank pain be managed with insulin.”
2. Oliguria and generalized edema 4. “Even with insulin, kidney damage is still a
3. Hematuria and proteinuria concern.”
4. Dysuria and hypotension 22. A patient diagnosed with sepsis from a UTI
16. You expect a patient in the oliguric phase is being discharged. What do you plan to
of renal failure to have a 24 hour urine output include in her discharge teaching?
less than: 1. Take cool baths
1. 200ml 2. Avoid tampon use
2. 400ml 3. Avoid sexual activity
3. 800ml 4. Drink 8 to 10 eight-oz glasses of water daily
4. 1000ml 23. You’re planning your medication teaching
17. The most common early sign of kidney for your patient with a UTI prescribed
disease is: phenazopyridine (Pyridium). What do you
include?
1. Sodium retention
2. Elevated BUN level 1. “Your urine might turn bright orange.”
3. Development of metabolic acidosis 2. “You need to take this antibiotic for 7 days.”
4. Inability to dilute or concentrate urine 3. “Take this drug between meals and at
bedtime.”
18. A patient is experiencing which type 4. “Don’t take this drug if you’re allergic
of incontinence if she experiences leaking to penicillin.”
urine when she coughs, sneezes, or lifts heavy
objects? 24. Which finding leads you to suspect acute
glomerulonephritis in your 32 y.o. patient?
1. Overflow
2. Reflex 1. Dysuria, frequency, and urgency
3. Stress 2. Back pain, nausea, and vomiting
4. Urge 3. Hypertension, oliguria, and fatigue
4. Fever, chills, and right upper quadrant pain
19. Immediately post-op after a radiating to the back
prostatectomy, which complications requires
priority assessment of your patient? 25. What is the priority nursing diagnosis with
your patient diagnosed with end-stage renal
1. Pneumonia disease?
2. Hemorrhage
3. Urine retention 1. Activity intolerance
4. Deep vein thrombosis 2. Fluid volume excess
3. Knowledge deficit
20. The most indicative test for 4. Pain
prostate cancer is:

1. A thorough digital rectal examination


2. Magnetic resonance imaging (MRI)
Reviewer

26. A patient with ESRD has an arteriovenous 2. Low-protein diet with a prescribed amount of
fistula in the left arm for hemodialysis. Which water
intervention do you include in his plan of care? 3. No protein in the diet and use of a salt
substitute
1. Apply pressure to the needle site upon
4. No restrictions
discontinuing hemodialysis
2. Keep the head of the bed elevated 45 31. After the first hemodialysis treatment, your
degrees patient develops a headache, hypertension,
3. Place the left arm on an arm board for at restlessness, mental confusion, nausea, and
least 30 minutes vomiting. Which condition is indicated?
4. Keep the left arm dry
1. Disequilibrium syndrome
27. Your 60 y.o. patient with pyelonephritis 2. Respiratory distress
and possible septicemia has had five UTIs over 3. Hypervolemia
the past two years. She is fatigued from lack of 4. Peritonitis
sleep, has lost weight, and urinates frequently
even in the night. Her labs show: sodium, 154 32. Which action is most important during
mEq/L; osmolarity 340 mOsm/L; glucose, 127 bladder training in a patient with a neurogenic
mg/dl; and potassium, 3.9 mEq/L. Which bladder?
nursing diagnosis is priority? 1. Encourage the use of an indwelling urinary
1. Fluid volume deficit related to osmotic catheter
diuresis induced by hyponatremia 2. Set up specific times to empty the bladder
3. Encourage Kegel exercises
2. Fluid volume deficit related to inability to
conserve water 4. Force fluids
3. Altered nutrition: Less than body 33. A patient with diabetes has had many renal
requirements related to hypermetabolic state calculi over the past 20 years and now has
4. Altered nutrition: Less than body chronic renal failure. Which substance must be
requirements related to catabolic effects of reduced in this patient’s diet?
insulin deficiency
1. Carbohydrates
28. Which sign indicated the second phase 2. Fats
of acute renal failure? 3. Protein
1. Daily doubling of urine output (4 to 5 L/day) 4. Vitamin C
2. Urine output less than 400 ml/day 34. What is the best way to check for patency
3. Urine output less than 100 ml/day of the arteriovenous fistula for hemodialysis?
4. Stabilization of renal function
1. Pinch the fistula and note the speed of filling
29. Your patient had surgery to form an on release
arteriovenous fistula for hemodialysis. Which 2. Use a needle and syringe to aspirate blood
information is important for providing care for from the fistula
the patient? 3. Check for capillary refill of the nail beds on
1. The patient shouldn’t feel pain during that extremity
initiation of dialysis 4. Palpate the fistula throughout its length to
2. The patient feels best immediately after the assess for a thrill
dialysis treatment 35. You have a paraplegic patient with renal
3. Using a stethoscope for auscultating the calculi. Which factor contributes to the
fistula is contraindicated development of calculi?
4. Taking a blood pressure reading on the
affected arm can cause clotting of the fistula 1. Increased calcium loss from the bones
2. Decreased kidney function
30. A patient with diabetes mellitus and renal 3. Decreased calcium intake
failure begins hemodialysis. Which diet is best 4. High fluid intake
on days between dialysis treatments?

1. Low-protein diet with unlimited amounts of


water
Reviewer

36. What is the most important nursing 41. Your patient returns from the operating
diagnosis for a patient in end-stage renal room after abdominal aortic aneurysm repair.
disease? Which symptom is a sign of acute renal failure?

1. Risk for injury 1. Anuria


2. Fluid volume excess 2. Diarrhea
3. Altered nutrition: less than body 3. Oliguria
requirements 4. Vomiting
4. Activity intolerance
42. Which cause of hypertension is the most
37. Frequent PVCs are noted on the cardiac common in acute renal failure?
monitor of a patient with end-stage renal
disease. The priority intervention is: 1. Pulmonary edema
2. Hypervolemia
1. Call the doctor immediately 3. Hypovolemia
2. Give the patient IV lidocaine (Xylocaine) 4. Anemia
3. Prepare to defibrillate the patient
43. A patient returns from surgery with an
4. Check the patient’s latest potassium level
indwelling urinary catheter in place and empty.
38. A patient who received a kidney transplant Six hours later, the volume is 120ml. The
returns for a follow-up visit to the outpatient drainage system has no obstructions. Which
clinic and reports a lump in her breast. intervention has priority?
Transplant recipients are:
1. Give a 500 ml bolus of isotonic saline
1. At increased risk for cancer due to 2. Evaluate the patient’s circulation and vital
immunosuppression caused signs
by cyclosporine (Neoral) 3. Flush the urinary catheter with sterile water
2. Consumed with fear after the life-threatening or saline
experience of having a transplant 4. Place the patient in the shock position, and
3. At increased risk for tumors because of the notify the surgeon
kidney transplant
4. At decreased risk for cancer, so the lump is 44. You’re preparing for
most likely benign urinary catheterization of a trauma patient and
you observe bleeding at the urethral meatus.
39. You’re developing a care plan with the Which action has priority?
nursing diagnosis risk for infection for your
patient that received a kidney transplant. A 1. Irrigate and clean the meatus before
goal for this patient is to: catheterization
2. Check the discharge for occult blood before
1. Remain afebrile and have negative cultures catheterization
2. Resume normal fluid intake within 2 to 3 days 3. Heavily lubricate the catheter
3. Resume the patient’s normal job within 2 to 3 before insertion
weeks 4. Delay catheterization and notify the doctor
4. Try to discontinue cyclosporine (Neoral) as
45. What change indicates recovery in a
quickly as possible
patient with nephritic syndrome?
40. You suspect kidney transplant rejection
when the patient shows which symptoms? 1. Disappearance of protein from the urine
2. Decrease in blood pressure to normal
1. Pain in the incision, general malaise, 3. Increase in serum lipid levels
and hypotension 4. Gain in body weight
2. Pain in the incision, general malaise,
and depression 46. Which statement correctly distinguishes
3. Fever, weight gain, and diminished urine renal failure from prerenal failure?
output 1. With prerenal failure, vasoactive substances
4. Diminished urine output and hypotension such as dopamine (Intropin) increase blood
pressure
2. With prerenal failure, there is less response
Reviewer

to such diuretics as furosemide (Lasix) 2. Answer: 2. “It burns when I pee.”


3. With prerenal failure, an IV isotonic saline
infusion increases urine output A common symptom of a UTI is dysuria. A
4. With prerenal failure, hemodialysis reduces patient with a UTI often reports frequent
voiding of small amounts and the urgency to
the BUN level
void.
47. Which criterion is required before a patient
can be considered for continuous peritoneal Option D: Urine that smells sweet is often
dialysis? associated with diabetic ketoacidosis.

1. The patient must be hemodynamically stable 3. Answer: 4. After painful urination is


2. The vascular access must have healed relieved, stop taking phenazopyridine.
3. The patient must be in a home setting Pyridium is taken to relieve dysuria because is
4. Hemodialysis must have failed provides an analgesic and anesthetic effect on
48. Polystyrene sulfonate (Kayexalate) is used the urinary tract mucosa. The patient can stop
in renal failure to: taking it after the dysuria is relieved.

1. Correct acidosis Option A: The urine may temporarily turn red or


2. Reduce serum phosphate levels orange due to the dye in the drug.
3. Exchange potassium for sodium Option B: The drug isn’t taken before voiding,
4. Prevent constipation from sorbitol use and is usually taken 3 times a day for 2 days.
49. Your patient has complaints of severe 4. Answer: 3. A 50 y.o. postmenopausal
right-sided flank pain, nausea, vomiting and woman
restlessness. He appears slightly pale and is
diaphoretic. Vital signs are BP 140/90 mmHg, Women are more prone to UTI’s
Pulse 118 beats/min., respirations 33 after menopause due to
breaths/minute, and temperature, 98.0F. reduced estrogen levels. Reduced estrogen
Which subjective data supports a diagnosis of levels lead to reduced levels of
renal calculi? vaginal Lactobacilli bacteria, which protect
against infection.
1. Pain radiating to the right upper quadrant
2. History of mild flu symptoms last week Options B, C, and D: Angina, asthma and
3. Dark-colored coffee-ground emesis fractures don’t increase the risk of UTI.
4. Dark, scanty urine output
5. Answer: 1. Check for kinks in the outflow
50. Immunosuppression following Kidney tubing.
transplantation is continued:
Tubing problems are a common cause of
1. For life outflow difficulties, check the tubing for kinks
2. 24 hours after transplantation and ensure that all clamps are open. Other
3. A week after transplantation measures include having the patient change
4. Until the kidney is not anymore rejected positions (moving side to side or sitting up),
applying gentle pressure over the abdomen, or
Answers and Rationale having a bowel movement.

6. Answer: 1. 15 minutes
1. Answer: 2. Costovertebral angle tenderness Dialysate should be infused quickly. The
and chills dialysate should be infused over 15 minutes or
Costovertebral angle tenderness, flank pain, less when performing peritoneal dialysis. The
and chills are symptoms of acute pyelonephritis. fluid exchange takes place over a period ranging
from 30 minutes to several hours.
Option A: Jaundice
7. Answer: 2. Avoid taking blood pressures in
indicates gallbladder or liver obstruction.
the arm with the fistula.
Option C: A burning sensation on urination is a
sign of lower urinary tract infection.
Reviewer

Don’t take blood pressure readings in the arm Narcotic analgesics are usually needed to
with the fistula because the compression could relieve the severe pain of renal calculi.
damage the fistula.
Options B and D: NSAIDS and salicylates are
Option A: IV lines shouldn’t be inserted in the used for their anti-inflammatory and antipyretic
arm used for hemodialysis. properties and to treat less severe pain.

Option C: Palpate pulses below the fistula. Option C: Muscle relaxants are typically used to
treat skeletal muscle spasms.
Option D: Lack of bruit or thrill should be
reported to the doctor. 14. Answer: 3. Invasive procedures

8. Answer: 2. Disequilibrium syndrome Invasive procedures such as catheterization can


introduce bacteria into the urinary tract. A lack
Disequilibrium syndrome is caused by a rapid of fluid intake could cause concentration of
reduction in urea, sodium, and other solutes
urine, but wouldn’t necessarily cause infection.
from the blood. This can lead to cerebral edema
and increased intracranial pressure (ICP). Signs 15. Answer: 3. Hematuria and proteinuria
and symptoms include headache, nausea,
restlessness, vomiting, confusion, twitching, and Hematuria and proteinuria indicate acute
seizures. glomerulonephritis. These finding result from
increased permeability of the glomerular
9. Answer: 2. Infuse normal saline solution membrane due to the antigen-antibody
reaction. Generalized edema is seen most often
Treatment includes administering normal saline in nephrosis.
or hypertonic normal saline solution because
muscle cramps can occur when the sodium and 16. Answer: 2. 400ml
water are removed to quickly during dialysis.
Reducing the rate of dialysis, not increasing it, Oliguria is defined as urine output of less than
may alleviate muscle cramps. 400ml/24hours.

10. Answer: 4. Keep fingernails short and clean 17. Answer: 2. Elevated BUN level

Calcium-phosphate deposits in the skin may Increased BUN is usually an early indicator of
cause pruritus. Scratching leads to excoriation decreased renal function.
and breaks in the skin that increase the 18. Answer: 3. Stress
patient’s risk of infection. Keeping fingernails
short and clean helps reduce the risk of Stress incontinence is an involuntary loss of a
infection. small amount of urine due to sudden increased
intra-abdominal pressure, such as with
11. Answer: 4. Strain all urine coughing or sneezing.
All urine should be strained through gauze or a 19. Answer: 2. Hemorrhage
urine strainer to catch stones that are passed.
The stones are then analyzed for composition. Hemorrhage is a potential complication.

Option A: Ambulation may help the movement Option A: Pneumonia may occur if the patient
of the stone down the urinary tract. doesn’t cough and deep breathe.

Option C: Encourage fluid to help flush the Option C: Urine retention isn’t a problem soon
stones out. after surgery because a catheter is in place.

12. Answer: 3. Recent sore throat Option D: Thrombosis may occur later if the
patient doesn’t ambulate.
The most common form of acute
glomerulonephritis is caused by group A beta- 20. Answer: 4. Prostate-specific antigen
hemolytic streptococcal infection elsewhere in
An elevated prostate-specific antigen level
the body.
indicates prostate cancer, but it can be falsely
13. Answer: 1. Narcotic analgesics elevated if done after the prostate gland is
manipulated.
Reviewer

Option A: A digital rectal examination should be 28. Answer: 1. Daily doubling of urine output
done as part of the yearly screening, and then (4 to 5 L/day)
the antigen test is done if the digital exam Daily doubling of the urine output indicates that
suggests cancer. the nephrons are healing. This means the
patient is passing into the second phase
Option B: MRI is used in staging the cancer. (dieresis) of acute renal failure.
21. Answer: 4. “Even with insulin, kidney 29. Answer: 4. Taking a blood pressure reading
damage is still a concern.” on the affected arm can cause clotting of the
Kidney damage is still a concern. Microvascular fistula Pressure on the fistula or the extremity
changes occur in both of the patient’s kidneys can decrease blood flow and precipitate
as a complication of the diabetes. Diabetic clotting, so avoid taking blood pressure on the
nephropathy is the leading cause of end-stage affected arm.
renal disease. The kidneys continue to produce 30. Answer: 2. Low-protein diet with a
urine until the end stage. Nephropathy occurs
prescribed amount of water
even with insulin management. The patient should follow a low-protein diet
22. Answer: 4. Drink 8 to 10 eight-oz glasses of with a prescribed amount of water. The patient
water daily requires some protein to meet metabolic needs.
Option C: Salt substitutes shouldn’t be used
Drinking 2-3L of water daily inhibits bacterial without a doctor’s order because it may contain
growth in the bladder and helps flush the potassium, which could make the patient
bacteria from the bladder. The patient should hyperkalemic. Option D: Fluid and
be instructed to void after sexual activity. protein restrictions are needed.

23. Answer: 1. “Your urine might turn bright 31. Answer: 1. Disequilibrium syndrome
orange.”
Disequilibrium occurs when excess solutes are
The drug turns the urine orange. It may be cleared from the blood more rapidly than they
prescribed for longer than 7 days and is usually can diffuse from the body’s cells into the
ordered three times a day after meals. vascular system.
Phenazopyridine is an azo (nitrogenous)
analgesic; not an antibiotic. 32. Answer: 2. Set up specific times to empty
the bladder
24. Answer: 3. Hypertension, oliguria, and
fatigue Instruct the patient with neurogenic bladder to
write down his voiding pattern and empty the
Mild to moderate HTN may result from sodium bladder at the same times each day.
or water retention and inappropriate renin
release from the kidneys. Oliguria and fatigue 33. Answer: 3. Protein
also may be seen. Other signs are proteinuria Because of damage to the nephrons, the kidney
and azotemia. can’t excrete all the metabolic wastes of
protein, so this patient’s protein intake must be
25. Answer: 2. Fluid volume excess restricted.
Fluid volume excess because the kidneys aren’t
removing fluid and wastes. The other diagnoses Options A, B, and D: A higher intake of carbs,
may apply, but they don’t take priority. fats, and vitamin supplements is needed to
ensure the growth and maintenance of the
26. Answer: 1. Apply pressure to the needle patient’s tissues.
site upon discontinuing hemodialysis
Apply pressure when discontinuing 34. Answer: 4. Palpate the fistula throughout
hemodialysis and after removing the its length to assess for a thrill
venipuncture needle until all the bleeding has The vibration or thrill felt during palpation
stopped. Bleeding may continue for 10 minutes ensures that the fistula has the desired
in some patients. turbulent blood flow. Pinching the fistula could
27. Answer: 2. Fluid volume deficit related to cause damage. Aspirating blood is a needless
inability to conserve water invasive procedure.
Reviewer

35. Answer: 1. Increased calcium loss from the 42. Answer: 2. Hypervolemia
bones
Acute renal failure causes hypervolemia as a
Bones lose calcium when a patient can no result of overexpansion of extracellular
longer bear weight. The calcium lost from bones fluid and plasma volume with the
form calculi, a concentration of mineral salts hypersecretion of renin. Therefore,
also known as a stone, in the renal system. hypervolemia causes hypertension.

36. Answer: 2. Fluid volume excess 43. Answer: 2. Evaluate the patient’s
circulation and vital signs
Kidneys are unable to rid the body of excess
fluids which results in fluid volume excess A total UO of 120ml is too low. Assess the
during ESRD. patient’s circulation and hemodynamic stability
for signs of hypovolemia. A fluid bolus may be
37. Answer: 4. Check the patient’s latest required, but only after further nursing
potassium level assessment and a doctor’s order.
The patient with ESRD may develop arrhythmias 44. Answer: 4. Delay catheterization and notify
caused by hypokalemia. the doctor
Option A: Call the doctor after checking the Bleeding at the urethral meatus is evidence that
patient’s potassium values. the urethra is injured. Because catheterization
Option B: Lidocaine may be ordered if the PVCs can cause further harm, consult with the
are frequent and the patient is symptomatic. doctor.

38. Answer: 1. At increased risk for cancer due 45. Answer: 1. Disappearance of protein from
to immunosuppression caused by cyclosporine the urine
(Neoral) With nephrotic syndrome, the glomerular
Cyclosporine suppresses the immune response basement membrane of the kidney becomes
to prevent rejection of the transplanted kidney. more porous, leading to loss of protein in the
The use of cyclosporine places the patient at urine. As the patient recovers, less protein is
risk for tumors. found in the urine.

39. Answer: 1. Remain afebrile and have 46. Answer: 3. With prerenal failure, an IV
negative cultures isotonic saline infusion increases urine output

The immunosuppressive activity of cyclosporine Prerenal failure is caused by such conditions as


places the patient at risk for infection, and hypovolemia that impairs kidney perfusion;
steroids can mask the signs of infection. The giving isotonic fluids improves urine output.
patient may not be able to resume normal fluid Vasoactive substances can increase blood
intake or return to work for an extended period pressure in both conditions.
of time and the patient may need cyclosporine 47. Answer: 1. The patient must be
therapy for life. hemodynamically stable
40. Answer: 3. Fever, weight gain, and
Hemodynamic stability must be established
diminished urine output before continuous peritoneal dialysis can be
Symptoms of rejection include fever, rapid started.
weight gain, hypertension, pain over the graft
48. Answer: 3. Exchange potassium for sodium
site, peripheral edema, and diminished urine
output. In renal failure, patients become hyperkalemic
because they can’t excrete potassium in the
41. Answer: 3. Oliguria urine. Polystyrene sulfonate acts to excrete
Urine output less than 50ml in 24 hours signifies potassium by pulling potassium into the bowels
oliguria, an early sign of renal failure. Anuria is and exchanging it for sodium.
uncommon except in obstructive renal
49. Answer: 4. Dark, scanty urine output
disorders.
Reviewer

Patients with renal calculi commonly have blood 4. Client’s support system and understanding of
in the urine caused by the stone’s passage lifestyle changes.
through the urinary tract. The urine appears
dark, tests positive for blood, and is typically 6. A client had a transurethral prostatectomy
for benign prostatic hypertrophy. He’s
scant.
currently being treated with a continuous
50. Answer: 1. For life. bladder irrigation and is complaining of an
increase in severity of bladder spasms. Which
of the interventions should be done first?
1. A client is complaining of severe flank and 1. Administer an oral analgesic
abdominal pain. A flat plate of the abdomen
2. Stop the irrigation and call the physician
shows urolithiasis. Which of the following 3. Administer a belladonna
interventions is important? and opium suppository as ordered by the
1. Strain all urine physician.
2. Limit fluid intake 4. Check for the presence of clots, and make
3. Enforce strict bed rest sure the catheter is draining properly.
4. Encourage a high calcium diet 7. A client is admitted with a diagnosis of
2. A client is receiving a radiation implant for hydronephrosis secondary to calculi. The
the treatment of bladder cancer. Which of the calculi have been removed and post
following interventions is appropriate? obstructive diuresis is occurring. Which of the
following interventions should be done?
1. Flush all urine down the toilet
2. Restrict the client’s fluid intake 1. Take vital signs every 8 hours
3. Place the client in a semi-private room 2. Weigh the client every other day
4. Monitor the client for signs and symptoms 3. Assess for urine output every shift
of cystitis 4. Monitor the client’s electrolyte levels.

3. A client has just received a renal transplant 8. A client has passed a renal calculus. The
and has started cyclosporine therapy to nurse sends the specimen to the laboratory so
prevent graft rejection. Which of the following it can be analyzed for which of the following
conditions is a major complication of this drug factors?
therapy? 1. Antibodies
1. Depression 2. Type of infection
2. Hemorrhage 3. Composition of calculus
3. Infection 4. Size and number of calculi
4. Peptic ulcer disease 9. Which of the following symptoms indicate
4. A client received a kidney transplant 2 acute rejection of a transplanted kidney?
months ago. He’s admitted to the hospital with 1. Edema, Nausea
the diagnosis of acute rejection. Which of the
2. Fever, Anorexia
following assessment findings would be 3. Weight gain, pain at graft site
expected? 4. Increased WBC count, pain with voiding
1. Hypotension 10. Adverse reactions of prednisone therapy
2. Normal body temperature
include which of the following conditions?
3. Decreased WBC count
4. Elevated BUN and creatinine levels 1. Acne and bleeding gums
2. Sodium retention and constipation
5. The client is to undergo kidney 3. Mood swings and increased temperature
transplantation with a living donor. Which of 4. Increased blood glucose levels and decreased
the following preoperative assessments is
wound healing.
important?
11. The nurse suspects that a client with
1. Urine output polyuria is experiencing water diuresis. Which
2. Signs of graft rejection
laboratory value suggests water diuresis?
3. Signs and symptoms of rejection
Reviewer

1. High urine specific gravity has hematuria and lower abdominal pain. To
2. High urine osmolarity determine further whether the pain is due to
3. Normal to low urine specific gravity bladder trauma, the nurse asks the client if the
4. Elevated urine pH pain is referred to which of the following
areas?
12. A client is diagnosed with prostate cancer.
Which test is used to monitor progression of 1. Shoulder
this disease? 2. Umbilicus
3. Costovertebral angle
1. Serum creatinine
4. Hip
2. Complete blood cell count (CBC)
3. Prostate-specific antigen (PSA) 18. The client complains of fever, perineal
4. Serum potassium pain, and urinary urgency, frequency,
and dysuria. To assess whether the client’s
13. A 27-year old client, who became problem is related to bacterial prostatitis, the
paraplegic after a swimming accident, is
nurse would look at the results of the prostate
experiencing autonomic dysreflexia. Which examination, which should reveal that the
condition is the most common cause of prostate gland is:
autonomic dysreflexia?
1. Tender, indurated, and warm to the touch
1. Upper respiratory infection 2. Soft and swollen
2. Incontinence 3. Tender and edematous with ecchymosis
3. Bladder distention 4. Reddened, swollen, and boggy.
4. Diarrhea
19. The nurse is taking the history of a client
14. When providing discharge teaching for a who has had benign prostatic hyperplasia in
client with uric acid calculi, the nurse should an the past. To determine whether the client
instruction to avoid which type of diet? currently is experiencing difficulty, the nurse
1. Low-calcium asks the client about the presence of which of
2. Low-oxalate the following early symptoms?
3. High-oxalate 1. Urge incontinence
4. High-purine 2. Nocturia
15. The client with urolithiasis has a history of 3. Decreased force in the stream of urine
chronic urinary tract infections. The nurse 4. Urinary retention
concludes that this client most likely has which 20. The client who has a cold is seen in the
of the following types of urinary stones? emergency room with inability to void.
1. Calcium oxalate Because the client has a history of BPH, the
2. Uric acid nurse determines that the client should be
3. Struvite questioned about the use of which of the
4. Cystine following medications?

16. The nurse is receiving in transfer from the 1. Diuretics


postanesthesia care unit a client who has had a 2. Antibiotics
percutaneous ultrasonic lithotripsy for 3. Antitussives
calculuses in the renal pelvis. The nurse 4. Decongestants
anticipates that the client’s care will 21. The nurse is preparing to care for the client
involve monitoring which of the following? following a renal scan. Which of the following
1. Suprapubic tube would the nurse include in the plan of care?
2. Urethral stent 1. Place the client on radiation precautions for
3. Nephrostomy tube 18 hours
4. Jackson-Pratt drain 2. Save all urine in a radiation safe container for
17. The client is admitted to the ER following a 18 hours
MVA. The client was wearing a lap seat belt 3. Limit contact with the client to 20 minutes
when the accident occurred. The client per hour.
Reviewer

4. No special precautions except to wear gloves resection syndrome. Which of the following
if in contact with the client’s urine. assessment data would indicate the onset of
this syndrome?
22. The client passes a urinary stone, and lab
analysis of the stone indicates that it is 1. Bradycardia and confusion
composed of calcium oxalate. Based on this 2. Tachycardia and diarrhea
analysis, which of the following would the 3. Decreased urinary output and bladder
nurse specifically include in the dietary spasms
instructions? 4. Increased urinary output and anemia

1. Increase intake of meat, fish, plums, and 27. The client is admitted to the hospital with
cranberries BPH, and a transurethral resection of the
2. Avoid citrus fruits and citrus juices prostate is performed. Four hours
3. Avoid green, leafy vegetables such as after surgery the nurse takes the client’s VS
spinach. and empties the urinary drainage bag. Which
4. Increase intake of dairy products. of the following assessment findings would
indicate the need to notify the physician?
23. The client returns to the nursing unit
following a pyelolithotomy for removal of a 1. Red bloody urine
kidney stone. A Penrose drain is in place. 2. Urinary output of 200 ml greater than intake
Which of the following would the nurse 3. Blood pressure of 100/50 and pulse 130.
include in the client’s postoperative care? 4. Pain related to bladder spasms.

1. Sterile irrigation of the Penrose drain 28. Which of the following symptoms is the
2. Frequent dressing changes around the most common clinical finding associated with
Penrose drain bladder cancer?
3. Weighing the dressings
4. Maintaining the client’s position on the 1. Suprapubic pain
2. Dysuria
affected side
3. Painless hematuria
24. The nurse is caring for a client following a 4. Urinary retention
kidney transplant. The client develops oliguria.
Which of the following would the nurse 29. A client who has been diagnosed with
anticipate to be prescribed as the treatment of bladder cancer is scheduled for an ileal
oliguria? conduit. Preoperatively, the nurse reinforces
the client’s understanding of the surgical
1. Encourage fluid intake procedure by explaining that an ileal conduit:
2. Administration of diuretics
3. Irrigation of Foley catheter 1. Is a temporary procedure that can be
reversed later.
4. Restricting fluids
2. Diverts urine into the sigmoid colon, where it
25. A week after kidney transplantation the is expelled through the rectum.
client develops a temperature of 101, 3. Conveys urine from the ureters to a stoma
the blood pressure is elevated, and the kidney opening in the abdomen.
is tender. The X-ray results the transplanted 4. Creates an opening in the bladder that allows
kidney is enlarged. Based on these assessment urine to drain into an external pouch.
findings, the nurse would suspect which of the
30. After surgery for an ileal conduit, the nurse
following?
should closely evaluate the client for the
1. Acute rejection occurrence of which of the following
2. Chronic rejection complications related to pelvic surgery?
3. Kidney infection
1. Peritonitis
4. Kidney obstruction
2. Thrombophlebitis
26. The client with BPH undergoes a 3. Ascites
transurethral resection of the prostate. 4. Inguinal hernia
Postoperatively, the client is receiving
continuous bladder irrigations. The nurse 31. The nurse is assessing the urine of a client
assesses the client for signs of transurethral who has had an ileal conduit and notes that
Reviewer

the urine is yellow with a moderate amount of worthlessness


mucus. Based on the assessment data, which 4. Disturbed Body Image related to creation of a
of the following nursing interventions would urinary diversion.
be most appropriate at this time?
36. The nurse teaches the client with a urinary
1. Change the appliance bag diversion to attach the appliance to a standard
2. Notify the physician urine collection bag at night. The most
3. Obtain a urine specimen for culture important reason for doing this is to prevent:
4. Encourage a high fluid intake
1. Urine reflux into the stoma
32. When teaching the client to care for an 2. Appliance separation
ileal conduit, the nurse instructs the client to 3. Urine leakage
empty the appliance frequently, primarily to 4. The need to restrict fluids
prevent which of the following problems?
37. The nurse teaches the client with an ileal
1. Rupture of the ileal conduit conduit measures to prevent a UTI. Which of
2. Interruption of urine production the following measures would be most
3. Development of odor effective?
4. Separation of the appliance from the skin
1. Avoid people with respiratory tract infections
33. The client with an ileal conduit will be using 2. Maintain a daily fluid intake of 2,000 to 3,000
a reusable appliance at home. The nurse ml
should teach the client to clean the appliance 3. Use sterile technique to change the appliance
routinely with what product? 4. Irrigate the stoma daily.

1. Baking soda 38. A client who has been diagnosed with


2. Soap calculi reports that the pain is intermittent and
3. Hydrogen peroxide less colicky. Which of the following nursing
4. Alcohol actions is most important at this time?

34. The nurse is evaluating the discharge 1. Report hematuria to the physician
teaching for a client who has an ileal conduit. 2. Strain the urine carefully
Which of the following statements indicates 3. Administer meperidine (Demerol) every 3
that the client has correctly understood the hours
teaching? Select all that apply. 4. Apply warm compresses to the flank area

1. “If I limit my fluid intake I will not have to 39. A client has a ureteral catheter in place
empty my ostomy pouch as often.” after renal surgery. A priority nursing action
2. “I can place an aspirin tablet in my pouch to for care of the ureteral catheter would be to:
decrease odor.”
3. “I can usually keep my ostomy pouch on for 3 1. Irrigate the catheter with 30 ml of normal
to 7 days before changing it.” saline every 8 hours
4. “I must use a skin barrier to protect my skin 2. Ensure that the catheter is draining freely
from urine.” 3. Clamp the catheter every 2 hours for 30
5. “I should empty my ostomy pouch of urine minutes.
when it is full.” 4. Ensure that the catheter drains at least 30 ml
an hour
35. A female client with a urinary diversion
tells the nurse, “This urinary pouch is 40. Which of the following interventions would
embarrassing. Everyone will know that I’m not be most appropriate for preventing the
normal. I don’t see how I can go out in public development of a paralytic ileus in a client who
anymore.” The most appropriate nursing has undergone renal surgery?
diagnosis for this patient is: 1. Encourage the client to ambulate every 2 to 4
hours
1. Anxiety related to the presence of urinary
diversion. 2. Offer 3 to 4 ounces of a carbonated beverage
2. Deficient Knowledge about how to care for periodically.
the urinary diversion. 3. Encourage use of a stool softener
3. Low Self-Esteem related to feelings of 4. Continue intravenous fluid therapy
Reviewer

41. The nurse is conducting a postoperative 46. Which of the following assessment data
assessment of a client on the first day after would most likely be related to a client’s
renal surgery. Which of the following findings current complaint of stress incontinence?
would be most important for the nurse to
1. The client’s intake of 2 to 3 L of fluid per day.
report to the physician?
2. The client’s history of three full-term
1. Temperature, 99.8 pregnancies
2. Urine output, 20 ml/hour 3. The client’s age of 45 years
3. Absence of bowel sounds 4. The client’s history of competitive swimming
4. A 2×2 inch area of serous sanguineous
drainage on the flank dressing. 47. The nurse is developing a teaching plan for
a client with stress incontinence. Which of the
42. Because a client’s renal stone was found to following instructions should be included?
be composed to uric acid, a low-purine,
alkaline ash diet was ordered. Incorporation of 1. Avoid activities that are stressful and
upsetting
which of the following food items into the
home diet would indicate that the client 2. Avoid caffeine and alcohol
understands the necessary diet modifications? 3. Do not wear a girdle
4. Limit physical exertion
1. Milk, apples, tomatoes, and corn
2. Eggs, spinach, dried peas, and gravy. 48. A client has urge incontinence. Which of
3. Salmon, chicken, caviar, and asparagus the following signs and symptoms would the
4. Grapes, corn, cereals, and liver. nurse expect to find in this client?

43. Allopurinol (Zyloprim), 200 mg/day, is 1. Inability to empty the bladder


prescribed for the client with renal calculi to 2. Loss of urine when coughing
take home. The nurse should teach the client 3. Involuntary urination with minimal warning
about which of the following side effects of 4. Frequent dribbling of urine
this medication? 49. A 72-year old male client is brought to the
1. Retinopathy emergency room by his son. The client is
2. Maculopapular rash extremely uncomfortable and has been unable
3. Nasal congestion to void for the past 12 hours. He has known for
some time that he has an enlarged prostate
4. Dizziness
but has wanted to avoid surgery. The best
44. The client has a clinic appointment method for the nurse to use when assessing
scheduled 10 days after discharge. Which for bladder distention in a male client is to
laboratory finding at that time would indicate check for:
that allopurinol (Zyloprim) has had a
1. A rounded swelling above the pubis.
therapeutic effect?
2. Dullness in the lower left quadrant
1. Decreased urinary alkaline phosphatase level 3. Rebound tenderness below the symphysis
2. Increased urinary calcium excretion 4. Urine discharge from the urethral meatus
3. Increased serum calcium level
50. During a client’s urinary
4. Decreased serum uric acid level
bladder catheterization, the bladder is emptied
45. When developing a plan of care for the gradually. The best rationale for the
client with stress incontinence, the nurse nurse’s action is that completely emptying an
should take into consideration that stress overdistended bladder at one time tends to
incontinence is best defined as the involuntary cause:
loss of urine associated with:
1. Renal failure
1. A strong urge to urinate 2. Abdominal cramping
2. Overdistention of the bladder 3. Possible shock
3. Activities that increase abdominal pressure 4. Atrophy of bladder musculature
4. Obstruction of the urethra
51. The primary reason for taping an
indwelling catheter laterally to the thigh of a
male client is to:
Reviewer

1. Eliminate pressure at the penoscrotal angle 4. When there is no drainage of urine and
2. Prevent the catheter from kinking in the irrigating solution
urethra
3. Prevent accidental catheter removal 57. A priority nursing diagnosis for the client
who is being discharged t home 3 days after a
4. Allow the client to turn without kinking the
catheter TURP would be:

52. The primary function of the prostate gland 1. Deficient fluid volume
2. Imbalanced Nutrition: Less than Body
is:
Requirements
1. To store underdeveloped sperm before 3. Impaired Tissue Integrity
ejaculation 4. Ineffective Airway Clearance
2. To regulate the acidity and alkalinity of the
environment for proper sperm development. 58. If a client’s prostate enlargement is caused
3. To produce a secretion that aids in the by a malignancy, which of the following blood
examinations should the nurse anticipate to
nourishment and passage of sperm
4. To secrete a hormone that stimulates the assess whether metastasis has occurred?
production and maturation of sperm 1. Serum creatinine level
53. The nurse is reviewing a medication history 2. Serum acid phosphatase level
of a client with BPH. Which medication should 3. Total nonprotein nitrogen level
be recognized as likely to aggravate BPH? 4. Endogenous creatinine clearance time

1. Metformin (Glucophage) 59. Steroids, if used following kidney


2. Buspirone (BuSpar) transplantation would cause which of the
3. Inhaled ipratropium (Atrovent) following side effects?
4. Ophthalmic timolol (Timoptic) 1. Alopecia
54. A client is scheduled to undergo a 2. Increase Cholesterol Level
transurethral resection of the prostate gland 3. Orthostatic Hypotension
(TURP). The procedure is to be done under 4. Increase Blood Glucose Level
spinal anesthesia. Postoperatively, the nurse 60. Mr. Roberto was readmitted to the
should be particularly alert for early signs of: hospital with acute graft rejection. Which of
1. Convulsions the following assessment finding would be
2. Cardiac arrest expected?
3. Renal shutdown 1. Hypotension
4. Respiratory paralysis 2. Normal Body Temperature
55. A client with BPH is being treated 3. Decreased WBC
with terazosin (Hytrin) 2 mg at bedtime. The 4. Elevated BUN and Creatinine
nurse should monitor the client’s: Answers and Rationale
1. Urinary nitrites
2. White blood cell count
3. Blood pressure 1. Answer: 1. Strain all urine
4. Pulse
Urine should be strained for calculi and sent to
56. A client underwent a TURP, and a large the lab for analysis.
three-way catheter was inserted into the
Option B: Fluid intake of three (3) to four (4) L is
bladder with continuous bladder irrigation. In
encouraged to flush the urinary tract and
which of the following circumstances would
prevent further calculi formation.
the nurse increase the flow rate of the
continuous bladder irrigation? Option C: Ambulation is encouraged to help
pass the calculi through gravity.
1. When the drainage is continuous but slow
2. When the drainage appears cloudy and dark Option D: A low-calcium diet is recommended
yellow to help prevent the formation of calcium calculi.
3. When the drainage becomes bright red
Reviewer

2. Answer: 4. Monitor the client for signs and postoperatively due to the immunosuppressant
symptoms of cystitis therapy. Rejection can occur postoperatively.

Cystitis is the most common adverse reaction of 6. Answer: 4. Check for the presence of clots,
clients undergoing radiation therapy; symptoms and make sure the catheter is draining
include dysuria, frequency, urgency, and properly.
nocturia. Clients with radiation implants require
a private room. Urine of clients with radiation Blood clots and blocked outflow if the urine can
implants for bladder cancer should be sent to increase spasms.
the radioisotopes lab for monitoring. It is Option A: Oral analgesics should be given if the
recommended that fluid intake be increased. spasms are unrelieved by the belladonna and
3. Answer: 3. Infection opium suppository.

Infection is the major complication to watch for Option B: The irrigation shouldn’t be stopped as
in clients on cyclosporine therapy because it’s long as the catheter is draining because clots
an immunosuppressive drug. will form.

Option A: Depression may occur Option C: A belladonna and opium suppository


should be given to relieve spasms but only after
posttransplantation but not because of
cyclosporine. assessment of the drainage.

Option B: Hemorrhage is a complication 7. Answer: 4. Monitor the client’s electrolyte


associated with anticoagulant therapy. levels.

Option D: Peptic ulcer disease is a complication Postobstructive diuresis seen in hydronephrosis


can cause electrolyte imbalances; lab values
of steroid therapy.
must be checked so electrolytes can be replaced
4. Answer: 4. Elevated BUN and creatinine as needed.
levels
Option A: VS should initially be taken every 30
In a client with acute renal graft rejection, minutes for the first 4 hours and then every 2
evidence of deteriorating renal function is hours.
expected.
Option B: The client’s weight should be taken
Option A: The client would most likely have daily to assess fluid status more closely.
acute hypertension.
Option C: Urine output needs to be assessed
Options B and C: The nurse would see elevated hourly.
WBC counts and fever because the body is
recognizing the graft as foreign and is 8. Answer: 3. Composition of calculus
attempting to fight it. The calculus should be analyzed for the
5. Answer: 4. Client’s support system and composition to determine appropriate
understanding of lifestyle changes. interventions such as dietary restrictions.

The client undergoing a renal transplantation Options A and D: The size and number of calculi
will need vigilant follow-up care and must aren’t relevant, and they don’t contain
adhere to the medical regimen. The client is antibodies.
most likely anuric or oliguric preoperatively but Option B: Calculi don’t result in infections.
postoperatively will require close monitoring of
urine output to make sure the transplanted 9. Answer: 3. Weight gain, pain at graft site
kidney is functioning optimally. While the client
Pain at the graft site and weight gain indicates
will always need to be monitored for signs and
the transplanted kidney isn’t functioning and
symptoms of infection, it’s most important
possibly is being rejected. Transplant clients
post-op will require close monitoring of urine
usually have edema, anorexia, fever, and
output to make sure the transplanted kidney is
nausea before transplantation, so those
functioning optimally. While the client will
symptoms may not indicate rejection.
always need to be monitored for signs and
symptoms of infection, it’s most important
Reviewer

10. Answer: 4. Increased blood glucose levels To control uric acid calculi, the client should
and decreased wound healing. follow a low-purine diet, which excludes high-
purine foods such as organ meats.
Steroid use tends to increase blood glucose
levels, particularly in clients with diabetes and Option A: A low-calcium diet decreases the risk
borderline diabetes. Steroids also contribute to for oxalate renal calculi.
poor wound healing and may cause acne, mood
swings, and sodium and water retention. Options B and C: Oxalate is an essential amino
Steroids don’t affect acid and must be included in the diet. A low-
thermoregulation, bleeding tendencies, or oxalate diet is used to control calcium or
constipation. oxalate calculi.

Options A, B, and C: Steroids don’t affect 15. Answer: 3. Struvite


thermoregulation, bleeding tendencies, or Struvite stones commonly are referred to as
constipation. infection stones because they form in urine that
11. Answer: 3. Normal to low urine specific is alkaline and rich in ammonia, such as with
gravity a urinary tract infection.

Option A: Calcium oxalate stones result from


Water diuresis causes low urine specific gravity,
low urine osmolarity, and a normal to elevated increased calcium intake or conditions that raise
serum sodium level. serum calcium concentrations.

Option A: High specific gravity Option B: Uric acid stones occur in clients with
indicates dehydration. Hypernatremia signals gout.
acidosis and shock. Option D: Cystine stones are rare and occur in
Option D: Elevated urine pH can result clients with a genetic defect that results in
from potassium deficiency, a high-protein diet, decreased renal absorption of the amino acid
or uncontrolled diabetes. cystine.

12. Answer: 3. Prostate-specific antigen (PSA) 16. Answer: 3. Nephrostomy tube

The PSA test is used to monitor prostate cancer A nephrostomy tube is put in place after a
progression; higher PSA levels indicate a percutaneous ultrasonic lithotripsy to treat
greater tumor burden. calculuses in the renal pelvis. The client may
also have a Foley catheter to drain urine
Option A: Serum creatinine levels may suggest produced by the other kidney. The nurse
blockage from an enlarged prostate. monitors the drainage from each of these tubes
and strains the urine to detect elimination of
Option B: CBC is used to diagnose anemia and the calculus fragments.
polycythemia.
17. Answer: 1. Shoulder
Option D: Serum potassium levels
identify hypokalemia and hyperkalemia. Bladder trauma or injury is characterized by
lower abdominal pain that may radiate to one
13. Answer: 3. Bladder distention of the shoulders. Bladder injury pain does not
Autonomic dysreflexia is a potentially life- radiate to the umbilicus, CV angle, or hip.
threatening complication of spinal cord injury, Options B, C, and D: Bladder injury pain do not
occurring from obstruction of the urinary
radiate to the umbilicus, CV angle, or hip.
system or bowel.
18. Answer: 1. Tender, indurated, and warm to
Option A: An URI could obstruct the respiratory
the touch
system, but not the urinary or bowel system.
The client with prostatitis has a prostate gland
Options B and D: Incontinence and diarrhea that is swollen and tender, but that is also warm
don’t result in obstruction of the urinary system to the touch, firm, and indurated. Systemic
or bowel, respectively. symptoms include fever with chills, perineal and
14. Answer: 4. High-purine low back pain, and signs of urinary tract
infection (which often accompany the disorder).
Reviewer

19. Answer: 3. Decreased force in the stream 24. Answer: 2. Administration of diuretics
of urine
To increase urinary output, diuretics and
Decreased force in the stream of urine is an osmotic agents are considered. The client
early sign of BPH. The stream later becomes should be monitored closely because fluid
weak and dribbling. The client then may overload can cause hypertension, congestive
develop hematuria, frequency, urgency, urge heart failure, and pulmonary edema.
incontinence, and nocturia. If untreated,
complete obstruction and urinary retention can Options A and D: Fluid intake would not be
occur. encouraged or restricted.

Option C: Irrigation of the Foley catheter will


20. Answer: 4. Decongestants
not assist in alleviating this oliguria.
In the client with BPH, episodes of urinary
retention can be triggered by certain 25. Answer: 1. Acute rejection
medications, such as Acute rejection most often occurs in the first
decongestants, anticholinergics, two (2) weeks after transplant. Clinical
and antidepressants. The client should be manifestations include fever, malaise, elevated
questioned about the use of these medications WBC count, acute hypertension, graft
if the client has urinary retention. Retention can tenderness, and manifestations of deteriorating
also be precipitated by other factors, such as renal function.
alcoholic beverages, infection, bedrest, and
becoming chilled. Option B: Chronic rejection occurs gradually
during a period of months to years.
21. Answer: 4. No special precautions except to
wear gloves if in contact with the client’s urine. Options C and D: Although kidney infection or
obstruction can occur, the symptoms presented
No specific precautions are necessary following in the question do not relate specifically to
a renal scan. Urination into a commode is these disorders.
acceptable without risk from the small amount
of radioactive material to be excreted. The 26. Answer: 1. Bradycardia and confusion
nurse wears gloves to maintain body secretion
Transurethral resection syndrome is caused by
precautions.
increased absorption of nonelectrolyte
22. Answer: 3. Avoid green, leafy vegetables irrigating fluid used during surgery. The client
such as spinach. may show signs of cerebral edema and
increased intracranial pressure such as
Oxalate is found in dark green foods such as increased blood pressure, bradycardia,
spinach. Other foods that raise urinary oxalate confusion, disorientation, muscle twitching,
are rhubarb, strawberries, chocolate, wheat visual disturbances, and nausea and vomiting.
bran, nuts, beets, and tea.
27. Answer: 3. Blood pressure of 100/50 and
23. Answer: 2. Frequent dressing changes pulse 130.
around the Penrose drain
A rapid pulse with a low blood pressure is a
Frequent dressing changes around the Penrose potential sign of excessive blood loss. The
drain is required to protect the skin against physician should be notified.
breakdown from urinary drainage. If urinary
drainage is excessive, an ostomy pouch may be Option A: Frank bleeding (arterial or venous)
placed over the drain to protect the skin. may occur during the first few days after
surgery.
Option A: A Penrose drain is not irrigated.
Option B: Some hematuria is usual for several
Option C: Weighing the dressings is not days after surgery. A urinary output of 200 ml of
necessary. greater than intake is adequate.
Option D: Placing the client on the affected side Options D: Bladder spasms are expected to
will prevent a free flow of urine through the occur after surgery.
drain.
28. Answer: 3. Painless hematuria
Reviewer

Painless hematuria is the most common clinical Option A: Because mucus in the urine is
finding in bladder cancer. Other symptoms expected, it is not necessary to change the
include frequency, dysuria, and urgency, but appliance bag or notify the physician.
these are not as common as the hematuria.
Option C: The mucus is not an indication of an
Options A and D: Suprapubic pain and urinary infection, so a urine culture is not necessary.
retention do not occur in bladder cancer.
32. Answer: 4. Separation of the appliance
29. Answer: 3. Conveys urine from the ureters from the skin
to a stoma opening in the abdomen.
If the appliance becomes too full, it is likely to
An ileal conduit is a permanent urinary pull away from the skin completely or to leak
diversion in which a portion of the ileum is urine onto the skin. A full appliance will not
surgically resected and one end of the segment rupture the ileal conduit or interrupt urine
is closed. The ureters are surgically attached to production. Odor formation has numerous
this segment of the ileum, and the open end of causes.
the ileum is brought to the skin surface on the
abdomen to form the stoma. The client must 33. Answer: 2. Soap
wear a pouch to collect the urine that A reusable appliance should be routinely
continually flows through the conduit. The cleaned with soap and water.
bladder is removed during the surgical
procedure and the ileal conduit is not 34. Answers: 3, 4. “I can usually keep my
reversible. ostomy pouch on for 3 to 7 days before
changing it.” and “I must use a skin barrier to
Option B: Diversion of the urine to the sigmoid protect my skin from urine.”
colon is called a ureter ileosigmoidostomy.
The client with an ileal conduit must learn self-
Option D: An opening in the bladder that allows care activities related to the care of the stoma
urine to drain externally is called a cystostomy. and ostomy appliances. The client should be
30. Answer: 2. Thrombophlebitis taught to increase fluid intake to about 3,000 ml
per day and should not limit intake. The ostomy
After pelvic surgery, there is an increased appliance should be changed approximately
chance of thrombophlebitis owing to the pelvic every 3 to 7 days and whenever a leak develops.
manipulation that can interfere with circulation A skin barrier is essential to protecting the skin
and promote venous stasis. from the irritation of the urine.

Option A: Peritonitis is a potential complication Option A: Adequate fluid intake helps to flush
of any abdominal surgery, not just pelvic mucus from the ileal conduit.
surgery.
Option B: An aspirin should not be used as a
Option C: Ascites is most frequently an method of odor control because it can be an
indication of liver disease. irritant to the stoma and lead to ulceration.

Option D: Inguinal hernia may be caused by an Option E: The ostomy pouch should be emptied
increase in abdominal pressure or a congenital when it is one-third to one-half full to prevent
weakness of the abdominal wall; a ventral the weight from pulling the appliance away
hernia occurs at the site of a previous from the skin.
abdominal surgery.
35. Answer: 4. Disturbed Body Image related
31. Answer: 4. Encourage a high fluid intake to the creation of a urinary diversion.

Mucus is secreted by the intestinal segment It is normal for clients to express fears and
used to create the conduit and is a normal concerns about the body changes associated
occurrence. The client should be encouraged to with a urinary diversion. Allowing the client
maintain a large fluid intake to help flush the time to verbalize concerns in a supportive
mucus out of the conduit. environment and suggest that she discuss these
concerns with people who have successfully
adjusted to ostomy surgery can help her begin
coping with these changes in a positive manner.
Reviewer

Options A and C: Although the client may be 39. Answer: 2. Ensure that the catheter is
anxious about this situation and self-esteem draining freely
may be diminished, the underlying problem is a
disturbance in body image. The ureteral catheter should drain freely
without bleeding at the site.
Option B: There are no data to support a
Option A: The catheter is rarely irrigated, and
diagnosis of Deficient Knowledge.
any irrigation would be done by the physician.
36. Answer: 1. Urine reflux into the stoma
Option C: The catheter is never clamped.
The most important reason for attaching the
appliance to a standard urine collection bag at Option D: The client’s total urine output
night is to prevent reflux into the stoma and (ureteral catheter plus voiding or Foley catheter
ureters, which can result in infection. output) should be 30 ml/hour.

Options B and C: Use of a standard collection 40. Answer: 1. Encourage the client to
bag also keeps the appliance from separating ambulate every two (2) to four (4) hours
from the skin and helps prevent urine leakage Ambulation stimulates peristalsis. A client with
from an overly full bag, but the primary purpose paralytic ileus is kept NPO until peristalsis
is to prevent reflux of urine. returns.
Option D: A client with a urinary diversion Option C: A stool softener will not stimulate
should drink 2000-3000 ml of fluid each day; it peristalsis.
would be inappropriate to suggest decreasing
fluid intake. Option D: Intravenous fluid infusion is a routine
postoperative order that does not have any
37. Answer: 2. Maintain a daily fluid intake of effect on preventing paralytic ileus.
2,000 to 3,000 ml
41. Answer: 2. Urine output, 20 ml/hour
Maintaining a fluid intake of 2,000 to 3,000
ml/day is likely to be effective in preventing UTI. The decrease in urinary output may indicate
A high fluid intake results in high urine output, inadequate renal perfusion and should be
which prevents urinary stasis and bacterial reported immediately. Urine output of 30
growth. ml/hour or greater is considered acceptable.

Option A: Avoiding people with respiratory tract Option A: A slight elevation in temperature is
infections will not prevent urinary tract expected after surgery.
infections.
Option C: Peristalsis returns gradually, usually
Option C: Clean, not sterile, technique is used to the second or third day after surgery. Bowel
change the appliance. sounds will be absent until then.

Option D: An ileal conduit stoma is not irrigated. Option D: A small amount of serous
sanguineous drainage is to be expected.
38. Answer: 2. Strain the urine carefully
42. Answer: 1. Milk, apples, tomatoes, and
Intermittent pain that is less colicky indicates corn
that the calculi may be moving along the urinary
tract. Fluids should be encouraged to promote Because a high-purine diet contributes to the
movement, and the urine should be strained to formation of uric acid, a low-purine diet is
detect the passage of the stone. advocated. An alkaline ash diet is also
advocated because uric acid crystals are more
Option A: Hematuria is to be expected from the likely to develop in acid urine. Foods that may
irritation of the stone. be eaten as desired in a low-purine diet include
Option C: Analgesics should be administered milk, all fruits, tomatoes, cereals, and corn.
when the client needs them, not routinely. Food allowed on an alkaline ash diet include
milk, fruits (except cranberries, plums, and
Option D: Moist heat to the flank area is helpful prunes), and vegetables (especially legumes and
when renal colic occurs, but it is less necessary green vegetables). Gravy, chicken, and liver are
as pain is lessened. high in purine.
Reviewer

43. Answer: 2. Maculopapular rash Option A: Emotional stressors do not cause


stress incontinence. It is caused most commonly
Allopurinol is used to treat renal calculi be relaxed pelvic musculature.
composed of uric acid. Side effects of
allopurinol include drowsiness, maculopapular Option C: Wearing Girdles is not
rash, anemia, abdominal pain, nausea, contraindicated.
vomiting, and bone marrow depression. Clients
should be instructed to report skin rashes and Option D: Although clients may be inclined to
limit physical exertion to avoid incontinence
any unusual bleeding or bruising.
episodes; they should be encouraged to seek
Options A, C, and D: Retinopathy, nasal treatment instead of limiting their activities.
congestion, and dizziness are not side effects of
allopurinol. 48. Answer: 3. Involuntary urination with
minimal warning
44. Answer: 4. Decreased serum uric acid level
A characteristic of urge incontinence is
By inhibiting uric acid synthesis, allopurinol involuntary urination with little or no warning.
decreases its excretion. The drug’s effectiveness Option A: The inability to empty the bladder is
is assessed by evaluating for a decreased serum urinary retention. Option B: Loss of urine when
uric acid concentration. coughing occurs with stress incontinence.
Option D: Frequent dribbling of urine is
Options A, B, and C: Allopurinol does not alter common in male clients after some types of
the level of alkaline phosphatase, not does it prostate surgery or may occur in women after
affect urinary calcium excretion or the serum
the development of
calcium level.
Option A: The inability to empty the bladder is
45. Answer: 3. Activities that increase urinary retention.
abdominal pressure
Option B: Loss of urine when coughing occurs
Stress incontinence is the involuntary loss of
with stress incontinence.
urine during such activities as coughing,
sneezing, laughing, or physical exertion. These Option D: Frequent dribbling of urine is
activities increase abdominal and detrusor common in male clients after some types of
pressure. prostate surgery or may occur in women after
the development of a vesicovaginal or
Option A: A strong urge to urinate is associated ureterovaginal fistula.
with urge incontinence.
49. Answer: 1. A rounded swelling above the
Option B: Overdistention of the bladder can pubis.
lead to overflow incontinence.
The best way to assess for a distended bladder
Option D: Obstruction of the urethra can lead to in either a male or female client is to check for a
urinary retention. rounded swelling above the pubis. The swelling
46. Answer: 2. The client’s history of three full- represents the distended bladder rising above
term pregnancies the pubis into the abdominal cavity.

The history of three pregnancies is most likely Option B: Dullness does not indicate a
the cause of the client’s current episodes of distended bladder.
stress incontinence. The client’s fluid intake, Option C: The client might experience
age, or history of swimming would not create tenderness or pressure above the symphysis.
an increase in intra-abdominal pressure.
Option D: No urine discharge is expected; the
47. Answer: 2. Avoid caffeine and alcohol urine flow is blocked by the enlarged prostate.
Client’s with stress incontinence are
50. Answer: 3. Possible shock
encouraged to avoid substances such as
caffeine and alcohol which are bladder irritants. Rapid emptying of an overdistended bladder
may cause hypotension and shock due to the
sudden change of pressure within the
abdominal viscera. Previously, removing no
Reviewer

more than 1,000 ml at one time was the 56. Answer: 3. When the drainage becomes
standard of practice, but this is no longer bright red
thought to be necessary as long as the
overdistended bladder is emptied slowly. The decision made by the surgeon to insert a
catheter after a TURP or prostatectomy
51. Answer: 1. Eliminate pressure at the depends on the amount of bleeding that is
penoscrotal angle expected after the procedure. During
continuous bladder irrigation after a TURP or
The primary reason for taping an indwelling prostatectomy, the rate at which the solution
catheter to a male client soothe penis is held in enters the bladder should be increased when
a lateral position to prevent pressure at the the drainage becomes brighter red. The color
penoscrotal angle. Prolonged pressure at the indicates the presence of blood. Increasing the
penoscrotal angle can cause a flow of irrigating solution helps flush the
ureterocutaneous fistula. catheter well so clots do not plug it.
52. Answer: 3. To produce a secretion that aids
Option B: There would be no reason to increase
in the nourishment and passage of sperm the flow rate when the return is continuous or
The prostate gland is located below the bladder when the return appears cloudy and dark
and surrounds the urethra. It serves one yellow.
primary purpose: to produce a secretion that Option D: Increasing the flow would be
aids in the nourishment and passage of sperm. contraindicated when there is no return of
53. Answer: 3. Inhaled ipratropium (Atrovent) urine and irrigating solution.

Atrovent is a bronchodilator, and its 57. Answer: 1. Deficient fluid volume


anticholinergic effects can aggravate urinary Deficient Fluid Volume is a priority diagnosis
retention. because the client needs to drink a large
Options A and B: Glucophage and BuSpar do not amount of fluid to keep the urine clear. The
affect the urinary system. urine should be almost without color. About
two (2) weeks after a TURP, when desiccated
Option D: Timolol does not have a systemic tissue is sloughed out, a secondary hemorrhage
effect. could occur. The client should be instructed to
call the surgeon or go to the ED if at any time
54. Answer: 4. Respiratory paralysis
the urine turns bright red.
If paralysis of vasomotor nerves in the
Option B: The client is not specifically at risk for
upper spinal cord occurs when spinal anesthesia
nutritional problems after a TURP. The client is
is used, the client is likely to develop respiratory
not specifically at risk for nutritional problems
paralysis. Artificial ventilation is required until
after a TURP.
the effects of the anesthesia subside.
Option C: The client is not specifically at risk for
Options A, B, and C: Convulsions, cardiac arrest,
impaired tissue integrity because there is no
and renal shutdown are not likely results of
external incision.
spinal anesthesia.
Option D: The client is not specifically at risk for
55. Answer: 3. Blood pressure
airway problems because the procedure is done
Terazosin (Hytrin) is an antihypertensive drug after spinal anesthesia.
that is also used in the treatment of BPH. Blood
58. Answer: 2. Serum acid phosphatase level
pressure must be monitored to ensure that the
client does not develop hypotension, syncope, The most specific examination to determine
or postural hypotension. The client should be whether a malignancy extends outside of the
instructed to change positions slowly. prostatic capsule is a study of the serum acid
phosphatase level. The level increases when a
Options A, B, and D: Urinary nitrites, white
malignancy has metastasized. The prostate-
blood cell count, and pulse rate are not affected
specific antigen (PSA) determination and a
by terazosin.
digital rectal examination are done when
screening for prostate cancer. Serum creatinine
Reviewer

level, total nonprotein nitrogen level, and treatment was yesterday. Which of the
endogenous creatinine clearance time give following interventions should be done first?
information about kidney function, not
1. Administer oxygen
Options A, C, and D: Serum creatinine level, 2. Elevate the foot of the bed
total nonprotein nitrogen level, and 3. Restrict the client’s fluids
endogenous creatinine clearance time give 4. Prepare the client for hemodialysis.
information about kidney function, not prostate
5. A client has a history of chronic renal failure
malignancy.
and received hemodialysis treatments three
59. Answer: 4. Increased Blood Glucose Level. times per week through an arteriovenous (AV)
fistula in the left arm. Which of the following
60. Answer: 4. Elevated BUN and Creatinine. interventions is included in this client’s plan of
1. Dialysis allows for the exchange of particles care?
across a semipermeable membrane by which 1. Keep the AV fistula site dry.
of the following actions? 2. Keep the AV fistula wrapped in gauze.
1. Osmosis and diffusion 3. Take the blood pressure in the left arm
2. Passage of fluid toward a solution with a 4. Assess the AV fistula for a bruit and thrill
lower solute concentration 6. Which of the following factors causes the
3. Allowing the passage of blood cells and nausea associated with renal failure?
protein molecules through it.
4. Passage of solute particles toward a solution 1. Oliguria
with a higher concentration. 2. Gastric ulcers
3. Electrolyte imbalances
2. A client is diagnosed with chronic renal 4. Accumulation of waste products
failure and told she must start hemodialysis.
Client teaching would include which of the 7. Which of the following clients is at greatest
following instructions? risk for developing acute renal failure?

1. Follow a high potassium diet 1. A dialysis client who gets influenza


2. Strictly follow the hemodialysis schedule 2. A teenager who has an appendectomy
3. There will be a few changes in your lifestyle. 3. A pregnant woman who has a
4. Use alcohol on the skin and clean it due fractured femur
to integumentary changes. 4. A client with diabetes who has a
heart catheterization
3. A client is undergoing peritoneal dialysis.
The dialysate dwell time is completed, and the 8. In a client in renal failure, which assessment
dwell clamp is opened to allow the dialysate to finding may indicate hypocalcemia?
drain. The nurse notes that the drainage has
stopped and only 500 ml has drained; the 1. Headache
amount the dialysate instilled was 1,500 ml. 2. Serum calcium level of 5 mEq/L
3. Increased blood coagulation
Which of the following interventions would be
done first? 4. Diarrhea

1. Change the client’s position. 9. A nurse is assessing the patency of an


2. Call the physician. arteriovenous fistula in the left arm of a client
who is receiving hemodialysis for the
3. Check the catheter for kinks or obstruction.
4. Clamp the catheter and instill more dialysate treatment of chronic renal failure. Which
at the next exchange time. finding indicates that the fistula is patent?

4. A client receiving hemodialysis treatment 1. Absence of bruit on auscultation of the


arrives at the hospital with a blood pressure of fistula.
200/100, a heart rate of 110, and a respiratory 2. Palpation of a thrill over the fistula
rate of 36. Oxygen saturation on room air is 3. Presence of a radial pulse in the left wrist
89%. He complains of shortness of breath, and 4. Capillary refill time less than 3 seconds in
+2 pedal edema is noted. His last hemodialysis the nail beds of the fingers on the left hand.
Reviewer

10. The client with chronic renal failure is at 15. The client with chronic renal failure returns
risk of developing dementia related to to the nursing unit following a hemodialysis
excessive absorption of aluminum. The nurse treatment. On assessment the nurse notes that
teaches that this is the reason that the client is the client’s temperature is 100.2. Which of the
being prescribed which of the following following is the most appropriate nursing
phosphate binding agents? action?

1. Alu-cap (aluminum hydroxide) 1. Encourage fluids


2. Tums (calcium carbonate) 2. Notify the physician
3. Amphojel (aluminum hydroxide) 3. Monitor the site of the shunt for infection
4. Basaljel (aluminum hydroxide) 4. Continue to monitor vital signs

11. The client newly diagnosed with chronic 16. The nurse is performing an assessment on
renal failure recently has begun hemodialysis. a client who has returned from the dialysis unit
Knowing that the client is at risk for following hemodialysis. The client is
disequilibrium syndrome, the nurse assesses complaining of a headache and nausea and is
the client during dialysis for: extremely restless. Which of the following is
the most appropriate nursing action?
1. Hypertension, tachycardia, and fever
2. Hypotension, bradycardia, and hypothermia 1. Notify the physician
3. restlessness, irritability, and generalized 2. Monitor the client
weakness 3. Elevate the head of the bed
4. Headache, deteriorating level of 4. Medicate the client for nausea
consciousness, and twitching.
17. The nurse is assisting a client on a low-
12. A client with chronic renal failure has potassium diet to select food items from the
completed a hemodialysis treatment. The menu. Which of the following food items, if
nurse would use which of the following selected by the client, would indicate an
standard indicators to evaluate the client’s understanding of this dietary restriction?
status after dialysis?
1. Cantaloupe
1. Potassium level and weight 2. Spinach
2. BUN and creatinine levels 3. Lima beans
3. VS and BUN 4. Strawberries
4. VS and weight.
18. The nurse is reviewing a list of components
13. The hemodialysis client with a left arm contained in the peritoneal dialysis solution
fistula is at risk for steal syndrome. The nurse with the client. The client asks the nurse about
assesses this client for which of the following the purpose of the glucose contained in the
clinical manifestations? solution. The nurse bases the response
knowing that the glucose:
1. Warmth, redness, and pain in the left hand.
2. Pallor, diminished pulse, and pain in the left 1. Prevents excess glucose from being removed
hand. from the client.
3. Edema and reddish discoloration of the left 2. Decreases risk of peritonitis.
arm 3. Prevents disequilibrium syndrome
4. Aching pain, pallor, and edema in the left 4. Increased osmotic pressure to produce
arm. ultrafiltration.

14. A client is admitted to the hospital and has 19. The nurse is preparing to care for a client
a diagnosis of early stage chronic renal failure. receiving peritoneal dialysis. Which of the
Which of the following would the nurse expect following would be included in the nursing
to note on assessment of the client? plan of care to prevent the major complication
associated with peritoneal dialysis?
1. Polyuria
2. Polydipsia 1. Monitor the client’s level of consciousness
3. Oliguria 2. Maintain strict aseptic technique
4. Anuria 3. Add heparin to the dialysate solution
4. Change the catheter site dressing daily
Reviewer

20. A client newly diagnosed with renal failure of chest pain. The client is tachycardic, pale,
is receiving peritoneal dialysis. During the and anxious. The nurse suspects air embolism.
infusion of the dialysate the client complains The nurse should:
of abdominal pain. Which action by the nurse
1. Continue the dialysis at a slower rate after
is most appropriate?
checking the lines for air
1. Slow the infusion 2. Discontinue dialysis and notify the physician
2. Decrease the amount to be infused 3. Monitor vital signs every 15 minutes for the
3. Explain that the pain will subside after the next hour
first few exchanges 4. Bolus the client with 500 ml of normal
4. Stop the dialysis saline to break up the air embolism.

21. The nurse is instructing a client 26. The nurse has completed client teaching
with diabetes mellitus about peritoneal with the hemodialysis client about self-
dialysis. The nurse tells the client that it is monitoring between hemodialysis treatments.
important to maintain the dwell time for the The nurse determines that the client best
dialysis at the prescribed time because of the understands the information given if the client
risk of: states to record the daily:

1. Infection 1. Pulse and respiratory rate


2. Hyperglycemia 2. Intake, output, and weight
3. Fluid overload 3. BUN and creatinine levels
4. Disequilibrium syndrome 4. Activity log

22. The client with acute renal failure has a 27. The client with an arteriovenous shunt in
serum potassium level of 5.8 mEq/L. The nurse place for hemodialysis is at risk for bleeding.
would plan which of the following as a priority The nurse would do which of the following as a
action? priority action to prevent this complication
from occurring?
1. Allow an extra 500 ml of fluid intake to dilute
the electrolyte concentration. 1. Check the results of the PT time as they are
2. Encourage increased vegetables in the diet ordered.
3. Place the client on a cardiac monitor 2. Observe the site once per shift
4. Check the sodium level 3. Check the shunt for the presence of a bruit
and thrill
23. The client with chronic renal failure who is 4. Ensure that small clamps are attached to the
scheduled for hemodialysis this morning is due
AV shunt dressing.
to receive a daily dose of enalapril (Vasotec).
The nurse should plan to administer this 28. The nurse is monitoring a client receiving
medication: peritoneal dialysis and nurse notes that a
client’s outflow is less than the inflow. Select
1. Just before dialysis actions that the nurse should take.
2. During dialysis
3. On return from dialysis 1. Place the client in good body alignment
4. The day after dialysis 2. Check the level of the drainage bag
3. Contact the physician
24. The client with chronic renal failure has an 4. Check the peritoneal dialysis system for kinks
indwelling catheter for peritoneal dialysis in
5. Reposition the client to his or her side.
the abdomen. The client spills water on the
catheter dressing while bathing. The nurse 29. The nurse assesses the client who has
should immediately: chronic renal failure and notes the following:
crackles in the lung bases, elevated blood
1. Reinforce the dressing pressure, and weight gain of 2 pounds in one
2. Change the dressing day. Based on these data, which of the
3. Flush the peritoneal dialysis catheter following nursing diagnoses is appropriate?
4. Scrub the catheter with povidone-iodine
1. Excess fluid volume related to the kidney’s
25. The client being hemodialyzed suddenly inability to maintain fluid balance.
becomes short of breath and complains 2. Increased cardiac output related to fluid
Reviewer

overload. 3. Pad the side rails of the bed


3. Ineffective tissue perfusion related to 4. Keep the client NPO.
interrupted arterial blood flow.
4. Ineffective Therapeutic Regimen 35. Aluminum hydroxide gel (Amphojel) is
prescribed for the client with chronic renal
Management related to lack of knowledge
about therapy. failure to take at home. What is the purpose of
giving this drug to a client with chronic renal
30. The nurse is caring for a hospitalized client failure?
who has chronic renal failure. Which of the
following nursing diagnoses are most 1. To relieve the pain of gastric hyperacidity
appropriate for this client? Select all that 2. To prevent Curling’s stress ulcers
apply. 3. To bind phosphorus in the intestine
4. To reverse metabolic acidosis.
1. Excess Fluid Volume
2. Imbalanced Nutrition; Less than Body 36. The nurse teaches the client with chronic
renal failure when to take the aluminum
Requirements
3. Activity Intolerance hydroxide gel. Which of the following
4. Impaired Gas Exchange statements would indicate that the client
5. Pain. understands the teaching?

31. What is the primary disadvantage of using 1. “I’ll take it every four (4) hours around the
peritoneal dialysis for long-term management clock.”
of chronic renal failure? 2. “I’ll take it between meals and at bedtime.”
3. “I’ll take it when I have a sour stomach.”
1. The danger of hemorrhage is high. 4. “I’ll take it with meals and bedtime snacks.”
2. It cannot correct severe imbalances.
3. It is a time consuming method of treatment. 37. The client with chronic renal failure tells
the nurse he takes magnesium hydroxide (milk
4. The risk of contracting hepatitis is high.
of magnesia) at home for constipation. The
32. The dialysis solution is warmed before use nurse suggests that the client switch to
in peritoneal dialysis primarily to: psyllium hydrophilic mucilloid (Metamucil)
because:
1. Encourage the removal of serum urea.
2. Force potassium back into the cells. 1. MOM can cause magnesium toxicity
3. Add extra warmth into the body. 2. MOM is too harsh on the bowel
4. Promote abdominal muscle relaxation. 3. Metamucil is more palatable
4. MOM is high in sodium
33. During the client’s dialysis, the nurse
observes that the solution draining from the 38. In planning teaching strategies for the
abdomen is consistently blood tinged. The client with chronic renal failure, the nurse
client has a permanent peritoneal catheter in must keep in mind the neurologic impact of
place. Which interpretation of this observation uremia. Which teaching strategy would be
would be correct? most appropriate?

1. Bleeding is expected with a permanent 1. Providing all needed teaching in one


peritoneal catheter extended session.
2. Bleeding indicates abdominal blood vessel 2. Validating frequently the client’s
damage understanding of the material.
3. Bleeding can indicate kidney damage. 3. Conducting a one-on-one session with the
4. Bleeding is caused by too-rapid infusion of client.
the dialysate. 4. Using videotapes to reinforce the material as
needed.
34. Which of the following nursing
interventions should be included in the client’s 39. The nurse helps the client with chronic
care plan during dialysis therapy? renal failure develop a home diet plan with the
goal of helping the client maintain adequate
1. Limit the client’s visitors nutritional intake. Which of the following diets
2. Monitor the client’s blood pressure would be most appropriate for a client with
chronic renal failure?
Reviewer

1. High carbohydrate, high protein 45. When caring for Mr. Roberto’s AV shunt on
2. High calcium, high potassium, high protein his right arm, you should:
3. Low protein, low sodium, low potassium
4. Low protein, high potassium 1. Cover the entire cannula with an elastic
bandage
40. A client with chronic renal failure has asked 2. Notify the physician if a bruit and thrill are
to be evaluated for a home continuous present
ambulatory peritoneal dialysis (CAPD) 3. User surgical aseptic technique when giving
program. The nurse should explain that the shunt care
major advantage of this approach is that it: 4. Take the blood pressure on the right arm
instead
1. Is relatively low in cost
2. Allows the client to be more independent Answers and Rationale
3. Is faster and more efficient than standard
peritoneal dialysis 1. Answer: 1. Osmosis and diffusion
4. Has fewer potential complications than Osmosis allows for the removal of fluid from the
standard peritoneal dialysis blood by allowing it to pass through the
41. The client asks whether her diet would semipermeable membrane to an area of high
change on CAPD. Which of the following would concentrate (dialysate), and diffusion allows for
passage of particles (electrolytes, urea, and
be the nurse’s best response?
creatinine) from an area of higher
1. “Diet restrictions are more rigid with CAPD concentration to an area of lower
because standard peritoneal dialysis is a more concentration.
effective technique.”
2. “Diet restrictions are the same for both CAPD Option B: Fluid passes to an area with a higher
and standard peritoneal dialysis.” solute concentration.
3. “Diet restrictions with CAPD are fewer than Option C: The pores of a semipermeable
with standard peritoneal dialysis because membrane are small, thus preventing the flow
dialysis is constant.” of blood cells and protein molecules through it.
4. “Diet restrictions with CAPD are fewer than
with standard peritoneal dialysis because CAPD 2. Answer: 2. Strictly follow the hemodialysis
works more quickly.” schedule

42. Which of the following is the most To prevent life-threatening complications, the
significant sign of peritoneal infection? client must follow the dialysis schedule.

1. Cloudy dialysate fluid Option A: The client should follow a low-


2. Swelling in the legs potassium diet because potassium levels
3. Poor drainage of the dialysate fluid increase in chronic renal failure.
4. Redness at the catheter insertion site
Option C: The client should know hemodialysis
43. The main indicator of the need for is time-consuming and will definitely cause a
hemodialysis is: change in current lifestyle.

1. Ascites Option D: Alcohol would further dry the client’s


2. Acidosis skin more than it already is.
3. Hypertension
3. Answer: 3. Check the catheter for kinks or
4. Hyperkalemia
obstruction.
44. To gain access to the vein and artery, an AV
The first intervention should be to check for
shunt was used for Mr. Roberto. The most
kinks and obstructions because that could be
serious problem with regards to the AV shunt
preventing drainage. After checking for kinks,
is:
have the client change position to promote
1. Septicemia drainage. Don’t give the next scheduled
2. Clot formation exchange until the dialysate is drained because
3. Exsanguination abdominal distention will occur, unless the
4. Vessel sclerosis output is within parameters set by the
Reviewer

physician. If unable to get more output despite Options B and C: A teenager who has an
checking for kinks and changing the client’s appendectomy and a pregnant woman with a
position, the nurse should then call the fractured femur isn’t at increased risk for renal
physician to determine the proper intervention. failure.

4. Answer: 1. Administer oxygen 8. Answer: 4. Diarrhea

Airway and oxygenation are always the first In renal failure, calcium absorption from the
priority. Because the client is complaining of intestine declines, leading to increased smooth
shortness of breath and his oxygen saturation is muscle contractions, causing diarrhea.
only 89%, the nurse needs to try to increase his
Option A: CNS changes in renal failure rarely
levels by administering oxygen.
include headache.
Option B: The foot of the bed may be elevated Option B: A serum calcium level of 5 mEq/L
indicates hypercalcemia.
to reduce edema, but this isn’t the priority.
Option C: As renal failure progresses, bleeding
Options C and D: The client is in pulmonary tendencies increase.
9. Answer: 2. Palpation of a thrill over the
edema from fluid overload and will need to be
fistula
dialyzed and have his fluids restricted, but the
first interventions should be aimed at the The nurse assesses the patency of the fistula by
immediate treatment of hypoxia. palpating for the presence of a thrill or
auscultating for a bruit.
5. Answer: 4. Assess the AV fistula for a bruit
and thrill Option A: The presence of a thrill and bruit
indicate patency of the fistula.
Assessment of the AV fistula for bruit and thrill
is important because, if not present, it indicates Options C and D: Although the presence of a
a non-functioning fistula. radial pulse in the left wrist and capillary refill
time less than 3 seconds in the nail beds of the
Option A: When not being dialyzed, the AV
fistula site may get wet. fingers on the left hand are normal findings;
Option B: Immediately after a dialysis they do not assess fistula patency.
treatment, the access site is covered with
adhesive bandages. 10. Answer: 2. Tums (calcium carbonate)
Option C: No blood pressures or venipunctures
should be taken in the arm with the AV fistula. Phosphate binding agents that contain
6. Answer: 4. Accumulation of waste products aluminum include Alu-caps, Basaljel, and
Amphojel. These products are made from
Although clients with renal failure can develop aluminum hydroxide. Tums are made from
stress ulcers, the nausea is usually related to calcium carbonate and also bind phosphorus.
the poisons of metabolic wastes that Tums are prescribed to avoid the occurrence of
accumulate when the kidneys are unable to dementia related to high intake of aluminum.
eliminate them. Phosphate binding agents are needed by the
client in renal failure because the kidneys
Options A and C: The client has electrolyte
cannot eliminate phosphorus.
imbalances and oliguria, but these don’t directly
cause nausea. 11. Answer: 4. Headache, deteriorating level of
consciousness and twitching.
7. Answer: 4. A client with diabetes who has a
heart catheterization Disequilibrium syndrome is characterized by
headache, mental confusion, decreasing level of
Clients with diabetes are prone to renal
consciousness, nausea, and vomiting, twitching,
insufficiency and renal failure. The contrast
and possible seizure activity. Disequilibrium
used for heart catheterization must be
syndrome is caused by rapid removal of solutes
eliminated by the kidneys, which further
from the body during hemodialysis. At the same
stresses them and may produce acute renal
time, the blood-brain barrier interferes with the
failure. A dialysis client already has end-stage
efficient removal of wastes from brain tissue. As
renal disease and wouldn’t develop acute renal
a result, water goes into cerebral cells because
failure.
of the osmotic gradient, causing brain swelling
and onset of symptoms. The syndrome most
Reviewer

often occurs in clients who are new to dialysis syndrome and appropriate treatments with
and is prevented by dialyzing for shorter times anticonvulsant medications
or at reduced blood flow rates. and barbiturates may be necessary to prevent a
life-threatening situation. The physician must be
12. Answer: 4. VS and weight. notified.
Following dialysis, the client’s vital signs are
17. Answer: 3. Lima beans
monitored to determine whether the client is
remaining hemodynamically stable. Weight is Lima beans (1/3 c) averages three (3) mEq per
measured and compared with the client’s serving.
predialysis weight to determine the
Option A: Cantaloupe (1/4 small)
effectiveness of fluid extraction.
Option B: Spinach (1/2 cooked)
Options A, B, and C: Laboratory studies are Option D: Strawberries (1 ¼ cups) are high
potassium foods and average 7 mEq per
done as per protocol but are not necessarily
serving.
done after the hemodialysis treatment has 18. Answer: 4. Increases osmotic pressure to
ended. produce ultrafiltration.
13. Answer: 2. Pallor, diminished pulse, and Increasing the glucose concentration makes the
pain in the left hand. solution increasingly more hypertonic. The
Steal syndrome results from vascular more hypertonic the solution, the greater the
insufficiency after the creation of a fistula. The osmotic pressure for ultrafiltration and thus the
client exhibits pallor and a diminished pulse greater amount of fluid removed from the client
distal to the fistula. The client also complains of during an exchange.
pain distal to the fistula, which is due to tissue 19. Answer: 2. Maintain strict aseptic
ischemia. technique
Option A: Warmth, redness, and pain more The major complication of peritoneal dialysis
likely would characterize a problem with is peritonitis. Strict aseptic technique is required
infection. in caring for the client receiving this treatment.
14. Answer: 1. Polyuria Although option D may assist in preventing
infection, this option relates to an external site.
Polyuria occurs early in chronic renal failure and
if untreated can cause severe dehydration. 20. Answer: 3. Explain that the pain will
Polyuria progresses to anuria, and the client subside after the first few exchanges
loses all normal functions of the kidney. Pain during the inflow of dialysate is common
Options B, C, and D: Oliguria and anuria are not during the first few exchanges because of
early signs, and polydipsia is unrelated to peritoneal irritation; however, the pain usually
chronic renal failure. disappears after 1 to 2 weeks of treatment. The
infusion amount should not be decreased, and
15. Answer: 4. Continue to monitor vital signs the infusion should not be slowed or stopped.

The client may have an elevated temperature 21. Answer: 2. Hyperglycemia


following dialysis because the dialysis machine
warms the blood slightly. If the temperature is An extended dwell time increases the risk of
elevated excessively and remains hyperglycemia in the client with diabetes
elevated, sepsis would be suspected, and a mellitus as a result of absorption of glucose
blood sample would be obtained as prescribed from the dialysate and electrolyte changes.
for culture and sensitivity purposes. Diabetic clients may require extra insulin when
receiving peritoneal dialysis.
16. Answer: 1. Notify the physician
22. Answer: 3. Place the client on a cardiac
Disequilibrium syndrome may be due to the monitor
rapid decrease in BUN levels during dialysis.
These changes can cause cerebral edema that The client with hyperkalemia is at risk for
leads to increased intracranial pressure. The developing cardiac dysrhythmias and cardiac
client is exhibiting early signs of disequilibrium
Reviewer

arrest. Because of this, the client should be weight daily. Ideally, the hemodialysis client
placed on a cardiac monitor. should not gain more than 0.5 kg of weight per
day.
Option A: Fluid intake is not increased because
it contributes to fluid overload and would not 27. Answer: 4. Ensure that small clamps are
affect the serum potassium level significantly. attached to the AV shunt dressing.

Option B: Vegetables are a natural source of An AV shunt is a less common form of access
potassium in the diet, and their use would not site but carries a risk of bleeding when it is used
be increased. because two ends of an external cannula are
tunneled subcutaneously into an artery and a
Option D: The nurse may also assess the sodium
vein and the ends of the cannula are joined. If
level because sodium is another electrolyte accidental connection occurs, the client could
commonly measured with the potassium level. lose blood rapidly. For this reason, small clamps
However, this is not a priority action at this are attached to the dressing that covers the
time. insertion site to use if needed.
23. Answer: 3. On return from dialysis Option B: The shunt site should be assessed at
Antihypertensive medications such as enalapril least every four hours.
are given to the client following hemodialysis. 28. Answer: 1, 2, 4, 5.
This prevents the client from becoming
hypotensive during dialysis and also from If outflow drainage is inadequate, the nurse
having the medication removed from the attempts to stimulate outflow by changing the
bloodstream by dialysis. No rationale exists for client’s position. Turning the client to the other
waiting a full day to resume the medication. side or making sure that the client is in good
This would lead to ineffective control of the body alignment may assist with outflow
blood pressure. drainage. The drainage bag needs to be lower
than the client’s abdomen to enhance gravity
24. Answer: 2. Change the dressing drainage. The connecting tubing and the
Clients with peritoneal dialysis catheters are at peritoneal dialysis system is also checked for
high risk for infection. A dressing that is wet is a kinks or twisting and the clamps on the system
conduit for bacteria for bacteria to reach the are checked to ensure that they are open. There
catheter insertion site. The nurse assures that is no reason to contact the physician.
the dressing is kept dry at all times. 29. Answer: 1. Excess fluid volume related to
Option A: Reinforcing the dressing is not a safe the kidney’s inability to maintain fluid balance.
practice to prevent infection in this Crackles in the lungs, weight gain, and elevated
circumstance. blood pressure are indicators of excess fluid
Option C: Flushing the catheter is not indicated. volume, a common complication in chronic
renal failure. The client’s fluid status should be
Option D: Scrubbing the catheter with monitored carefully for imbalances on an
povidone-iodine is done at the time of ongoing basis.
connection or disconnecting of peritoneal
dialysis. 30. Answer: 1, 2, 3.

25. Answer: 2. Discontinue dialysis and notify Appropriate nursing diagnoses for clients with
the physician chronic renal failure include excess fluid volume
related to fluid and sodium retention;
If the client experiences air embolus during imbalanced nutrition, less than body
hemodialysis, the nurse should terminate requirements related to anorexia, nausea, and
dialysis immediately, notify the physician, and vomiting; and activity intolerance related
administer oxygen as needed. to fatigue.

26. Answer: 2. Intake, output, and weight Options D and E: The nursing diagnoses of
impaired gas exchange and pain are not
The client on hemodialysis should monitor fluid
commonly related to the chronic renal failure.
status between hemodialysis treatments by
recording intake and output and measuring
Reviewer

31. Answer: 3. It is a time-consuming method 35. Answer: 3. To bind phosphorus in the


of treatment. intestine

The disadvantages of peritoneal dialysis in the A client in renal failure develops


long-term management of chronic renal failure hyperphosphatemia that causes a
is that requires large blocks of time. corresponding excretion of the body’s calcium
stores, leading to renal osteodystrophy. To
Options A and D: The risk of hemorrhage or decrease this loss, aluminum hydroxide gel is
hepatitis is not high with PD. prescribed to bind phosphates in the intestine
Option B: PD is effective in maintaining a client’s and facilitate their excretion.
fluid and electrolyte balance. Option A: Gastric hyperacidity is not necessarily
32. Answer: 1. Encourage the removal of a problem associated with chronic renal failure.
serum urea. Options B and D: Antacids will not prevent
The main reason for warming the peritoneal Curling’s stress ulcers and do not affect
dialysis solution is that the warm solution helps metabolic acidosis.
dilate peritoneal vessels, which increases urea 36. Answer: 4. “I’ll take it with meals and
clearance. bedtime snacks.”
Options B and D: The warmed solution does not Aluminum hydroxide gel is administered to bind
force potassium into the cells or promote the phosphates in ingested foods and must be
abdominal muscle relaxation. given with or immediately after meals and
Option C: Warmed dialyzing solution also snacks.
contributes to client comfort by preventing
Option A: There is no need for the client to take
chilly sensations, but this is a secondary reason it on a 24-hour schedule.
for warming the solution.
Options B and C: It is not administered to treat
33. Answer: 2. Bleeding indicates abdominal hyperacidity in clients with CRF and therefore is
blood vessel damage not prescribed between meals.
Because the client has a permanent catheter in 37. Answer: 1. MOM can cause magnesium
place, blood tinged drainage should not occur. toxicity
Persistent blood tinged drainage could indicate
damage to the abdominal vessels, and the Magnesium is normally excreted by the kidneys.
physician should be notified. When the kidneys fail, magnesium can
accumulate and cause severe neurologic
Option C: The bleeding is originating in the problems.
peritoneal cavity, not the kidneys.
Option B: MOM is harsher than Metamucil, but
Option D: Too rapid infusion of the dialysate can magnesium toxicity is a more serious problem.
cause pain.
Option C: A client may find both MOM and
34. Answer: 2. Monitor the client’s blood Metamucil unpalatable.
pressure
Option D: MOM is not high in sodium.
Because hypotension is a complication of
peritoneal dialysis, the nurse records intake, 38. Answer: 2. Validating frequently the
and output, monitors VS, and observes the client’s understanding of the material.
client’s behavior.
Uremia can cause decreased alertness, so the
Option A: The nurse also encourages visiting nurse needs to validate the client’s
and other diversional activities. comprehension frequently.

Options C and D: A client on PD does not need Option A: Because the client’s ability to
to be placed in bed with padded side rails or concentrate is limited, short lesions are most
kept NPO. effective.
Reviewer

Option C: If family members are present at the


sessions, they can reinforce the material.

Option D: Written materials that the client can


review are superior to videotapes, because the
clients may not be able to maintain alertness
during the viewing of the videotape.

39. Answer: 3. Low protein, low sodium, low

Option A: CAPD is costly and must be done


daily.

Option D: Side effects and complications are


similar to those of standard peritoneal dialysis.

41. Answer: 3. “Diet restrictions with CAPD are


fewer than with standard peritoneal dialysis
because dialysis is constant.”

Dietary restrictions with CAPD are fewer than


those with standard peritoneal dialysis because
dialysis is constant, not intermittent. The
constant slow diffusion of CAPD helps prevent
accumulation of toxins and allows for a more
liberal diet. CAPD does not work more quickly,
but more consistently. Both types of peritoneal
dialysis are effective.

42. Answer: 1. Cloudy dialysate fluid

Cloudy drainage indicates bacterial activity in


the peritoneum. Other signs and symptoms of
infection are fever, hyperactive bowel sounds,
and abdominal pain.

Option B: Swollen legs may be indicative


of congestive heart failure.

Option C: Poor drainage of dialysate fluid is


probably the result of a kinked catheter.

Option D: Redness at the insertion site indicates


local infection, not peritonitis. However, a local
infection that is left untreated can progress to
the peritoneum.

43. Answer: 4. Hyperkalemia

44. Answer: 3. Exsanguination

45. Answer: 3. User surgical aseptic technique


when giving shunt care

You might also like